clinmedjoel-140412001503-phpapp01

November 8, 2017 | Author: Aldwin Bagtas | Category: Headache, Vertigo, Human Anatomy, Human Head And Neck, Medical Specialties
Share Embed Donate


Short Description

ClinMed Joel...

Description

 

1  

                                       

                                                       

By  Medical  Students       For  Medical  Students  

2014     “This  is  not  intended  to  be  your  reviewer.  Your  best  reviewer  is  of  Barbara  Bates,     your  best  handout  is  your  ear,  and  your  best  teacher  is  yourself”        

 

2  

    CLINICAL  NEUROLOGY  by  Dra  Rosales                     3     HEAD,  EYES,  EARS,  NOSE,  THROAT  by  Dra  Solis                   8     EXTREMITIES  by  Dr  Paulino                       34     CHEST  and  LUNGS,  BREAST  by  Dra  Lee                     56     CARDIOLOGY  by  Dra  Deduyo                     65     ABDOMEN  by  Dra  Cortez                         76                 Please  be  guided  that  answers  are  coming  from  medical  students;  do  not  rely.       AKO  NA  MISMO  NAGSASABING  MALI  MALI  UNG  IBANG  MGA  SAGOT  DITO  KASI  AYAW  MAKINIG  NG  IBA  NA  NAG  EENCODE   SABING  WAG  PALITAN  UNG  SAGOT.         -­‐  N                                                            

  NEUROLOGY   1. Gait  ataxia  and  hypotonia  will  be  seen  in  a  patient  with  a  lesion  in  the   a. Rostral  vermis   b.Posterior  lobe          c.Caudal  vermis   d.All       e.  A  and  C   2. Manifestation  of  vermian  lesion   a. Dysarthria   b.  Scanning  speech   c.  Nystagmus   d.All   e.A  and  c   3. Manifestation  of  cerebellar  dysfunction   a. Hypotonia   b.  Decomposition  of  movement     c.  Mild  aesthenia    d.All       e.  A  and  C   4. Test  for  arm  dystaxia   a. Wrist  tapping  test   b.  Arm  pulling  test   c.  Thigh  patting  test     d.All     e.A  and  C   5. True  in  a  patient  with  cerebellar  hemisphere  infarct  in  the  right   a. Nystagmus   b.  Limb  ataxia     C.  Dysmetria  L     d.  All     e.  A  and  B   6. Superficial  sensation  routinely  examined   a. Romberg     b.  Asteriognosis     c.  Joint  position     d.  Pain                e.  C  and  D   7. Manifestation  of  polyneuropathy   a. Symetrical  distal  weakness     b.  Areflexia   c.  Preferential  sensory  loss  in  proximal  limb       d.  A  and  B       e.  All   8. From  medial  to  lateral  (CTLS)  segmented  arrangement  of  fibers  in  the  spinal  cord  is  seen  in  the     a. Fascicular  cuneatus/  gracilis   b.  Lateral  cortico  spinal       c.  Spinothalamic   d.  All   e.  B  and  C   9. TRUE  during  testing  of  sensory  function,     a. Usually  done  with  patient’s  eyes  closed   b. Should  compare  cornified  vs.  Non  cornified  areas   c. Should  examine  symmetrical  dermatomal  area   d. All   e. A  and  C   10. Pins  and  needles  sensation   a. Dysaesthesia   b.  Paresthesia   c.  Alodynia     e.  Analgesia   11. Biceps  reflex  is  subserved     a. C2  –  c3     b.C4-­‐c5     c.  C5-­‐c6    d.  C7-­‐c8   12. Variations  of  babinski   a. Snout  reflex   c.  Hoffman’s   d.  Chaddock’s     e.All   b. B  and  C   13. Examination  of  motor  function   a. Look  for  involuntary  movements   c. Inspect  muscle  size   b. Look  for  coordinated  performance  of   d. all   motor  acts   14. TRUE  of  spastic  gate:   a. Narrow  base  of  support   b. Foot  plantar  flexed  and  everted   c. Leg  externally  rotated  at  the  hip   d. All   e. A  and  C   15. Upper  motor  neuron  paralysis:   a. Muscles  affected  in  groups   e. A  and  C   b. Rigidity   c. Babinski   d. All  

3  

 

4   16. 17. 18. 19. 20.

                                                                                           

Fundoscopy   D   Nystagmus   C   Say  EGG  A     Open  mouth   B   Corneal  blink  reflex    

        B  

  a.  Cranial  nerve  from  medulla     b.  Cranial  nerve  from  the  pons   c.  Cranial  nerve  from  the  midbrain     d.  Cranial  nerve  from  the  supra  tentorial  

 

5  

A. MATCHING  TYPE:   1. A  stoke  patient  can  open  his  eyes,  moans  and  flexes  on  painful  stimulation  has  a  Glasgow  coma  scale  of:   A. 3/15   B. 6/15   C. 10/15   D. 12/15   2. The  main  objective  of  doing  deep  tendon  reflex  is  to:   A. Differentiate  whether  the  lesion  is  UMN  or  LMN   B. Differentiate  whether  the  lesion  is  brain  stem  or  spinal  cord   C. Differentiate  whether  the  lesion  is  anterior  horn  cell  or  peripheral  nerve   D. All  of  the  above   3. Babinski  is  not  seen  in:   A. Frontal  lobe  tumor   B. Brainstem  stroke   C. Spinal  cord   D. Diabetic   compression   neuropathy   4. A  high  stepped,  slapping  gate  is  usually  secondary  to:   A. Parkinson’s  disease     B. Posterior  column   C. UMN  Lesion   D. Gait  apraxia   lesion   5. Paralysis  of  upward  gaze  is  usually  secondary  to:   A. Optic  nerve  lesion   B. Optic  chiasm  lesion   C. Pineal  lesion   D. Pituitary  Lesion   6. Not  a  feature  of  metabolic  encephalopathy   A. Pupils  are  equal  and  reactive   C. Common  occurrence  of  movement  abnormality   B. Severe  mental  status  alteration   D. Presence  of  lateralizing  sign   7. Fatigue  with  exercise  is  usually  seen  in:   A. Neuropathy   B. Myopathy   C. Neuromuscular  function  disorder   D. UMN  lesion   8. A  unilateral,  dilated,  non  –  reactive  pupil  in  a  comatose  pate  will  indicate:     A. Metabolic  encephalopathy   C. Pontine  hemorrhage   B. Temporal  bone  herniation   D. Drug  overdose   9. A  patient  complaining  of  weakness  that  he  can  only  move  his  extremities  against  gravity  is  graded  as:   A. 1/5   B. 2/5   C. 3/5   D. 4/5   10. Mental  status  examination  is  a  test  for  the  integrity  of:   A. Cerebral  cortex   B. Cerebellum   C. Brainstem   D. Cranial  nerves   11. An  optic  chiasm  lesion  would  cause:   A. Failure  of   B. Paralysis  of  upward   C. Bitemporal  Hemianopsia   D. All  of  the  above   convergence   gaze   12. Lesion  in  the  cavernous  sinus  would  involve  the  following  cranial  nerves,  except:   A. CN  III   B. CN  IV   C. CN  V   D. CN  VI   13. Weakness  of  knee  extension  is  caused  by  a  lesion  of:   A. Femoral  nerve   B. Peroneal  nerve   C. Sciatic  Nerve   D. Popliteal  nerve   14. Features  of  upper  motor  neuron,  except:   A. Spastic   B. (+)  Babinski   C. Atrophy   D. None  of  the  above   15. Foot  drop  is  secondary  to  a  lesion  in  the:   A. Femoral  nerve   B. Peroneal  nerve   C. Lateral  cutaneous   D.   nerve   16. The  first  cranial  nerve  to  be  affected  by  increased  intracranial  pressure  is:   A. CN  III   B. CN  IV   C. CN  VI   D. CN  VI   17. Signs  of  peripheral  nerve  lesion,  except:   A. Atrophy   B. Fasciculations   C. Spasticity   D. Hyporeflexia   18. Neurologic  examination  in  a  comatose  patient  includes  the  following,  except:   A. Fundoscopy   B. Pupillary  light  reflex   C. Mental  status   D. Cerebellar  examination   examination   19. Signs  and  symptoms  of  myopathy  includes  the  following,  except:   A. Proximal  weakness   B. Atrophy   C. Hyperreflexia   D. None  of  the  above   20. The  most  reliable  sign  of  UMN  lesion:   A. Hyperreflexia   B. Atrophy  and  fasciculation   C. (+)  Babinski   D. Spasticity   21. Presence  of  cheery  red  spots  seen  in  the  retina  by  fundoscopy  is  seen  in:   A. Tay  Sach’s  disease   C. Inreacrania  hemorrhage  

 

6  

B. Subarachnoid  hemorrhage   D. Von  Recklinghausen’s  disease   22. Presence  of  café  au  lait  spots  seen  in  fundoscopy  is  secondary  to   A. Tay  Sach’s  disease   C. Inreacrania  hemorrhage   B. Subarachnoid  hemorrhage   D. Von  Recklinghausen’s  disease   23. Ataxia  is  seen  in  the  following  lesions,  except:   A. Dorsal  column   B. Cerebellum   C. Spinocerebellar  tract   D. None  of  the  above   24. Movement  disorder  associated  with  basal  ganglia  lesions,  except:   A. Dysmetria   B. dystonia   C. Bradykinesia   D. Tremor         25. In  the  lesion  of  the  superior  orbital  fissure  the  following  are  true,  except:     A. Weakness  of  the  lateral  movement  of  the  eye   B. Weakness  of  the  medial  movement  of  the  eye   C. Weakness  of  the  superior  and  inferior  movement  of  the  eye   D. None  of  the  above     B. ASSOCIATION  TYPE   A  –  If  1,2,3  are  correct   D  –  Only  4  is  correct   B  –  If  1  and  3  are  correct   E  –  if  all  are  correct   C  –  If  2  and  4  are  correct     E  26.  Neurologic  examination  is  mandatory  if  the  patient  complains  of:   1. Loss  of  consciousness   2. Weakness   3. Urinary  incontinence   4. Dementia   E    27.Sensory  examination  includes  the  following:   1. Crude  touch   2. Position   3. Vibration   4. Temperature   B    28.  Bitemporal  hemianopsia  is  secondary  to:   1. Pituitary  adenoma   2. Brainstem  lesion   3. Uncal  herniation   4. Metabolic  encepalopathy   E    29.  The  appearance  of  the  following  reflexes  would  indicate  frontal  lobe  lesion     1. Palmomental   2. Sucking   3. Grasp   4. Babinski   A    30.  Babinski  could  be  present  if  you  have  lesions  of  the:   1. Supratentorial   2. Posterior  fossa   3. Spinal   4. Anterior  horn  cell   A    31.  Neurologic  examination  in  a  comatose  patient  includes:   1. Light  reflex   2. Fundoscopy   3. Mental  status   4. Cerebellar  exam   C    32.  Retinal  hemorrhage  could  be  seen  in:   1. Optic  neuritis   2. Malignant   3. Cerebral  infarction   4. Raptured  aneurysm   hypertension   B      33.  Mental  status  examination  includes:   1. Level  of  consciousness     2. Memory   3. Mood   4. Position   B      34.  Fundoscopy  is  valuable  if  you  are  entering:   1. Increased  intracranial  pressure   2. Demyelinating  disorder   3. Raptures  aneurysm   4. Cerebral  infarction   C      35.  Signs  and  symptoms  of  myopathy   1. Absent  sensory  changes   2. Proximal  weakness   3. Atriphy   4. Hyporeflexia   A      36.  Movement  disorders  associated  with  basal  ganglia  lesion   1. Athetosis   2. Ballismus   3. Parkinson’s  disease   4. Dystonia   C      37.  Small,  pinpoint  pupils  are  associated  with:   1. Amphetamines   2. Opiate  overdose   3. Uncal  herniation   4. Pontine  lesion   overdose   D      38.  Large,  dilated  pupils  are  associated  with:   1. Cocaine  use   2. Heroine  use   3. Opiate  overdose   4. Amphetamine  overdose   E      39.  Horner’s  syndrome:   1. Mitotic  pupils   2. Ptosis   3. Facial  anhydrosis   4. Dilated  pupils  

  D      40.  Melkensson  syndrome   1. Painful  edema   2. Caused  by  herpes  zoster  virus   3. Pitting  edema     C. MATCHING  TYPE   I   41. Phonation   A. Cranial  I   G   42. Numbness  of  the  face   B. Cranial  II   B   43. Visual  acuity   C. Cranial  III   E   44. Clenching  of  teeth   D. Cranial  IV   K   45. Shoulder  shrug   E. Cranial  V   L   46. Tongue  atrophy   F. Cranial  VI   C   47. Convergence   G. Cranial  VII       H. Cranial  VIII       I. Cranial  IX       J. Cranial  X       K. Cranial  XI       L. Cranial  XII     B   48. CN  II  –  III   A. Corneal  reflex   49. CN  V  –  VII   B. Pupillary  light  reflex   A   50. CN   I X   –   X   C. Gag  reflex   C                                                              

7  

4.

Scrotal  tongue  

 

8  

HEENT     1.  Unilateral  headache  that  can  be  localized  behind  the  eyes  –  CLUSTER   2.  Enlarged  blind  spot  occurs  in  a  condition  affecting  the  optic  nerve   3.  Rhinoscopy  (ndi  dapat  tamaan)  –  NASAL  SEPTUM   4.  polyps  –    MEDIAL  MEATUS   5.  Family  history  –  MIGRAINE   6.  Hyperthyroidism  –  GRAVE’S  DISEASE   7.  button  like  –CHANCRE  IN  SYPHILIS   8.  maplike  –GEOGRAPHICAL   9.  caused  by  deficiency  in  riboflavin  and  niacin  –  SMOOTH  TONGUE   10.  ear  pull  (adults)  –  UP  &  BACK   11.  unilateral  painless  –  RETINAL  WALL   12.  examining  the  oropharynx  use  a  tongue  depressor  –  DISTAL  HALF  OF  TONGUE   13.  white  optic  disc  and  tiny  vessel  are  absent  –  OPTIC  ATROPHY   14.  fissured  tongue  –  APPEARED  WITH  INCREASING  AGE   15.  nutritional  deficiency  (cold  sore)  –  ANGULAR  CHEILITIS   16.  caused  by  trauma  –  SUBCONJUNCTIVAL  HEMORRHAGE   17.  tonsilar  LN  pulsation       a.  carotid  artery     b.  ext.  Jugular  Vein     c.  lymph  adenopathy     d.  bruit     18.  convergence  test  –   19.  Tug  test  painful    in  –  OTITIS  EXTERNA     ADDITIONAL:     • Otitis  Externa  –  pale,  moist  ,  narrow   •

Retracted  eardrum  



Soft  nodule  in  the  thyroid  gland  –  GRAVE’s  



Behind  the  eye  –  Cluster  



Riboflavin  def.  and  chemotx.  –  smooth  



Exposure  to  sumlight  –  lip  carcinoma  



Factor  to  carcinoma  –  Actinic  Cheilitis  



Worsen  in  noisy  environmanet  –  sensorineural  hearing  loss(SNHL)  



Presbyopic  –  better  seen  further  away   Central    loss  



Button  like  infection  –  Angular  cheilitis  



Ill  fitting  dentures  



Fissure  tongue  –  increasing  age  



Sign  of  lip  canar?  –  Actinic  cheilitis  

    1.  Hold  target  at  the  midline  and  at  eye  level  gradually  moving  the  target  toward  the  bridge  of  the  nose.     a.  Lid  lag     b.  Convergence     c.  Accommodation     d.  Confrontation     2.  For  the  test  above     a.  This  is  normally  maintained  at  a  distance  of  2-­‐3  inches  from  the  nasal  bridge  of  the  nose     b.  Watch  for  the  appearance  of  white  sclera  between  the  iris  and  the  upper  lid.     c.  Usually  a  person  sees  both  sets  of  fingers  at  the  same  time.     d.  There  is  pupillary  constriction  in  the  opposite  eye     3.  There  is  poor  convergence  in     a.  Hypothyroidism     b.  Hypertension     c.  Hyperthyroidism     d.  Diabetes  mellitus     4.  Absent  red  reflex  suggest     a.  Normal  eye     b.  Artificial  eye     c.  Papilledema     d.  Hyperthyroidism     5.  External  auditory  canal  is  often  swollen  ,  narrowed  moist,  pale  and  tender.  It  may  be  reddened.  This  is     a.  Chronic  otitis  externa     b.  Acute  otitis  externa     c.  Acute  purulent  otitis  media     d.  acute  otitis  media     6.  This  is  not  a  special  eye  technique  for  eye  examination     a.  Nasolacrimal  duct  obstruction     b.  Inspection  of  the  upper  palpebral  conjunctiva     c.  For  assessing  protruding  eyes     d.  Extraocular  muscle  test   7.    Inspection  of  the  nasal  cavity  through  the  anterior  naris  us  usually  limited  to  the  following:     a.  Vestibule     b.  Superior  turbinates     c.  Sphenoid  sinus     d.  Frontal  sinus   8.  Spinning  sensation  is  accompanied  by  nystagmus  and  ataxia     a.  Vertigo     b.  Dizziness     c.  Tinnitus     d.  Disequilibrium   9.  Retracted  tympanic  membrane     a.  More  conical     b.  Seen  on  Acute  suppurative  otitis  media     c.  Loss  of  bony  landmarks     d.  Accentuated  bony  landmarks     10.  Local  cause  of  nosebleeding  

9  

    a.  Flying     b.  Hypertension     c.  Nose  Picking     d.  Leukemia     e.  English     11.  Deacrease  facial  mobility  and  characteristic  stare  (Parkinson’s  disease)   12.  Head  is  elongated  with  bony  prominence  of  the  forehead,  nose  and  lower  jaw  (Acromegaly)   13.  swelling  usually  appears  first  in  the  eyes  and  in  the  morning  (Nephrotic  syndrome)   14.  hair  is  dry,  coarse  and  sparse  with  periorbital  edema.  Lateral  eyebrows  thin  (Myxedema)   15.  Red  cheeks,  hirsutism  and  “moonface”  (Cushing’s  syndrome)   16.  May  accompany  lipid  disorders  (Xanthelasma)   17.  Tearing  is  prominent.  Nasolacrimal  duct  obstruction  is  also  noted  (Dacryocyctitis)   18.  Usually  points  inside  the  lid  rather  that  the  lid  margin  (Chalazion)   19.  A  painful,  tender  red  infection  in  a  gland  at  the  margin  of  the  eyelids  (Sty)   20.  Drooping  of  eyelids  (Ptosis)     Q:  Unilateral  headache  that  can  be  localized  behind  the  eye   A.  Cluster     Q:  Enlarge  Blind  spot  occurs  in  a  condition  affecting  optic  nerve   A:       Q:  Rhinoscopy  (hindi  dapat  tamaan)   A:  Nasal  septum     Q:  Polyps   A:  Medial  Meatus     Q:  Family  History   A:  Migraine     Q:  Hyperthyroidism   A:  Grave’s  disease     Q:  Button  like   A:  Chancre  syphilis     Q:  Map-­‐like   A:  Geographical     Q:  Caused  by  a  deficiency  in  riboflavin  and  Niacin   A:  smooth  Tongue     Q:  Ear  pull  (adult)   A:  Up  and  back     Q:  Unilateral,  painless   A:  Retinal  Wall     Q:  Examining  oropharynx,  use  tongue  depressor:   A:  Distal  half  of  tongue    

10  

 

11  

Q:  White  optic  disc  and  tiny  vessel  are  absent   A:  Optic  Atropy     Q:  Fissured  tongue   A:  Appeared  with  increasing  age         • Subarachnoid  Hemorrhage  -­‐  nausea  and  vomiting,  possible  loss  of  consciousness,  neck  pain     •

Ptosis  -­‐  cause  myasthenia  gravis,  damage  to  the  oculomotor  nerve  (CN  III),  damage  to  the  sympathetic  nerve   supply  (  Horner's  syndrome)  



Ectropion  -­‐  the  margin  of  the  lower  lid  is  turned  outward,  exposing  the  palpebral  conjunctiva  



Inflammation  of  the  Lacrimal  Sac  (Dacryocystitis)  -­‐  swelling  between  the  lower  eyelid  and  the  nose  



Horner's  Syndrome-­‐  small  affected  pupil,  reacts  briskly  to  light  and  near  effort,    ptosis  present,  loss  of   sweating  on  forehead,  heterochromia  



Argyll-­‐Robertson  pupils  -­‐  small,  irregular  pupils  that  accomodate  but  do  not  react  to  light  indicate  CNS  syphilis  



Normal  arterial  wall  is  transparent;  Normal  light  reflex  is  narrow  



Silver  wiring  -­‐  occasionally  a  portion  of  a  narrowed  artery  develops  such  as  an  opaque  wall  that  has  no  blood   is  visible  within  it.  



Microaneurysms  -­‐  tiny,  round  red  spots  seen  commonly  but  not  exclusively  in  and  around  the  macular  area;   minute  dilatations  of  very  small  retinal  vessels,  but  the  vascular  connections  are  too  small  to  be  seen   opthalmoscopically  



Neovascularizations  -­‐  formation  of  new  blood  vessels;  more  numerous,  more  tortuous,  and  narrower  than   other  blood  vessels  in  the  area  and  form  disorderly  looking  red  arcades  



Hypertensive  Retinopathy  -­‐  marked  arteriolar  venous  crossing  changes  are  seen,  copper  wiring  of  the   arterioles  is  present.  Cotton  wool  spot  is  seen  just  superior  to  the  disc.  



Nonproliferative  Retinopathy  (  Moderately  severe)  -­‐  tiny  red  dots/  microaneurysms  



Proliferative  Retinopathy  (  Neovascularization)  -­‐  new  preretinal  vessels  arising  on  the  disc  extening  across  the   disc  margins.  Visual  acuity  is  still  normal,  but  risk  for  visual  loss  is  high  

  •

Keloid  -­‐  a  firm,  nodular,  hypertrophic  mass  of  scar  tissue  (binding)  extending  beyond  the  area  of  injury  



Tophi  -­‐  deposit  of  uric  acid  crystals  characteristic  of  chronic  tophaceous  gout  



Cutaneous  cyst/  Sebaceous  cyst  -­‐  a  dome  shaped  lump  in  the  dermis  forms  a  benign  closed  firm  sac   attached  to  the  dermis  



Rheumatoid  Nodules  -­‐  small  lump  on  the  helix/antihelix  and  additional  nodules  elsewhere  on  the  hands   along  the  surface  of  the  ulna  distal  to  the  elbow  

 

12   •

Acute  Otitis  Media  with  Purulent  Effusion  -­‐  caused  by  bacterial  infection  earache,  fever  and  hearing  loss.   Hearing  loss  is  of  the  conductive  type  



Sensorineural  loss  -­‐  weber's  test:  sound  lateralizes  to  good  ear  



Angular  Cheilitis  -­‐  softening  of  the  skin  at  the  angles  of  the  mouth,  fissuring  



Chancre  of  Syphilis  -­‐  appear  on  the  lip,  firm  button-­‐like  lesion  



Large  Normal  Tonsils  -­‐  normal  tonsils  may  be  enlarged;  protrude  medially  beyond  the  pillars  and  even  to   the  midline  



Diptheria  -­‐  dull  red,  gray  exudate  (pseudomembrane)  is  present  on  the  uvula,  pharynx  and  tongue  



Koplik's  spots  -­‐  early  sign  of  measles,  small  white  specks  that  resembles  grains  of  salt  



Acute  Necrotizing  Ulcerative  Gingivitis  -­‐  ulcers  develop  in  the  interdental  papilla  



Hutchinson's  teeth  -­‐  sides  of  these  teeth  show  normal  contours;  sides,  shaping  of  the  teeth  are  unaffected  



Smooth  tongue  (Atrophic  Glossitis)  -­‐  lost  its  papillae,  deficiency  in  riboflavin,  niacin,  folic  acid,  Vit.  B12,   pyridoxine,  iron  



Apthous  ulcer  (Canker  sores)  -­‐  painful,  round/oval  ulcer  that  is  white/yellowish  gray  and  surrounded  by  a   halo  of  reddened  mucosa  



Diffuse  Enlargement  -­‐  endemic  goiter  



Hyperthyroidism  -­‐  tachycardia  

1. 2.

Enlarged  skull  may  signify:  Hydrocephalus  or  Paget’s  disease  of  Bone   20/200  vision  meaning:  at  20  ft.,  the  patient  can  read  print  that  a  person  with  normal  vision  could  read  at   200  feet.   Absence  of  a  red  reflex:  Cataract  (opacity  of  lens),  detached  retina,  retinoblastoma   Light  rays  from  a  distance  focus  on  the  anterior  of  retina:  Myopia   Light  rays  from  a  distance  focus  on  the  posterior  of  retina:  Hyperopia   Loss  of  venous  pulsation  in  pathologic  conditions  like  head  trauma,  meningitis,  or  mass  lesions  may  be  an   early  sign  of:  Elevated  ICP   Canal  is  swollen,  narrowed,  moist,  pale,  tender,  reddened:  Acute  Otitis  externa   Unilateral  Conductive  hearing  loss:  Sound  is  heard  in  the  Impaired  ear   Unilateral  Sensorineural  Hearing  loss:  Sound  is  heard  in  the  good  ear   Conductive  hearing  loss:  BC  >  AC   Mucosa  is  reddened  and  swollen:  Viral  rhinitis   Mucosa  is  pale,  bluish,  or  red:  Allergic  rhinitis   Submental  Lymph  node  (maytanong  bout  sa  Submental  LN)   Basic  landmark  for  palpating  Thyroid  gland:  Thyroid  cartilage  and  Cricoid  cartilage??   Nausea,  Vomiting:  Migraine/Subarachnoid   Sudden  movements  of  the  head  may  be  associated  with:  Brain  tumor   Spinning  sensation:  Vertigo   Left  Homonymous  Hemianopsia:  Right  Optic  Radiation   Damage  to  Oculomotor  nerve:  Ptosis   Eye  no  longer  drains  satisfactorily:  Ectropion  

 

 

 

3. 4. 5. 6. 7. 8. 9. 10. 11. 12. 13. 14. 15. 16. 17. 18. 19.

 

13   20. Bilateral  Exopthalmos:  Grave’s,  hyperthyroidism   Unilateral  exophthalmos:  Grave’s  dse/  Tumor/  Inflammation  of  the  orbit   21. Painful,  Tender,  Red  infection  of  the  margin  of  the  eyelid:  Stye   22. Pupil  is  large,  regular  and  usually  unilateral;  reaction  to  light  is  severely  reduced/  slowed  or  absent:  Adie’s   pupil  (Tonic  pupil)   23. Loss  of  venous  pulsation;  disc  vessels  more  visible,  more  numerous:  papilledema   24. Arteries  show  areas  of  focal/generalized  narrowing:  hypertension   25. Arteries  become  full  and  somewhat  tortuous;  Inc  light  reflex:  Copper  wiring   26. Presence  of  cotton-­‐wool  spot:  Hypertensive  Retinopathy   27. May  tanong  on  page  267,  di  ko  maalala  pero  meron   28. Softening  of  the  skin  at  the  angles  of  the  mouth  followed  by  fissuring:  Angular  Cheilitis   29. Firm  lesion  on  the  lip:  Chancre  of  Syphilis   30. Reddened  throat  without  exudate:  Pharyngitis   31. Smooth  tongue  that  has  lost  its  papillae:  Atrophic  glossitis   32. Headache  is  severe  and  sudden  onset:  Subarachnoid  hemorrhage/  Acute  meningitis   33. Sudden  unilateral  visual  loss  is  pinless:  Retinal  detachment/  retinal  vein  occlusion/  central  retinal  artery   occlusion/  vitreous  hemorrhage/  macular  degeneration   *If  visual  loss  is  painful:  corneal  ulcer/  uveitis/  acute  glaucoma/  optic  neuritis   34. Bilateral  and  painless  visual  loss:  d/t  cholinergics,  anticholinergics  and  steroids/  Chemical,  radiation   exposure   35. People  having  trouble  understanding  speech;  noisy  environment  makes  hearing  worse:  Sensorineural  loss   36. Local  cause  of  epistaxis:  Trauma  (nose  picking)  –  most  common   37. Enlarged  blind  spot:  Glaucoma/  Optic  neuritis/  Papilledema  

    HEENT   1. In  primary  position,  R  eye    deviates  laterally  but  cannot  move  medially   a. R  lateral  rectus  palsy   b. R  oblique  rectus  palsy   c. R  medial  rectus  palsy   d. R  superior  rectus  palsy     2. Renal  artery  of  HTN-­‐  focal  narrowing       3. A  portion  of  a  narrowed  artery  develops  such  an  opaque  wall  that  no  blood    is  visible  with  in  it   a. silver  artery  or  silver  wire  artery   b. copper  wire   c. normal  artery   d. retinal  sclerosis     4. most  important  attribute  for  head  ache.    A.  Chronologic  pattern     B.  Quality     C.  Location     D.  Timing     5. Holding  a  pencil  and  moving  toward  the  bridge  of  the  nose  (ganyan  hung  thought  nung  question,  mahaba   kasi  yung  question  e)  Ans:  Convergence  test    

 

14   6. a. b. c. d. 7. 8. 9. 10. 11. 12.

13.

14. 15.

16. 17. 18. 19. 20.                  

True  about  the  test  above   5-­‐8  cm  distance  from  the  nose….   …   …   …               Inspection  of  anterior  nares  is  limited  to:   a. Vestibule   b. Sphenoid  sinus   c. Sup.  Turbinate   d. Frontal  sinus     Spinning  sensation  with  nystagmus  and  ataxia     a.  Vertigo    b.  Dizziness    c.  Tinnitus     d.  Dysequilibrium     ans.  A.  VERTIGO?       Retracted  tympanic  membrane….     Local  cause  of  nose  bleeding     A.  Flying     B.Hypertension     C.  Nose  picking   D.  Leukemia     In  opthalmoscopic  examination,-­‐The  view  is  limited  to  posterior    structure     Benign  lesion  associated  with  antibiotic  therapy-­‐  hairy  tongue   Examination  of  LN  is  done  by-­‐  palpation   Basic  landmark  of  thyroid  gland-­‐  Cricothyroid  

  Test  I:  Choose  the  BEST  answer   1.  In  opthalmoscopic  Examination   a.  The  view  is  limited  to  posterior  structure   b.  Pheripheral  structures  can  be  evaluated  in  the  absence  of  mydriatic  drops   c.  (+)3  or  (+)4  diopters  will  allow  you  to  see  the  anterior  structures  clearly   d.  Red-­‐orange  reflex  is  normally  not  visualize  first     2.  A  (-­‐)  lens  is  used  in   a.  Hyperopic  eyes   b.  Aphakic  eyes   c.  Myopic  eye   d.  Astigmatism     3.  Physical  sign  of  retrosternal  goiter   a.  Venous  engorgement   b.  Tender  thyroid   c.  Thyroid  bruit   d.  Regional  lymphnode  enlargement     4.  Examinatin  of  the  lymphnode  is  primarily  by   a.  Inspection   b.  Palpation   c.  Auscultation   d.  X-­‐ray     5.  Basic  landmark  for  thyroid  gland  examination   a.  Cricoid  cartilage   b.  Thyroid  cartilage   c.  Trachea   d.  Sternocleidomastoid     6.  Primary  lesion  from  posterior  2/3  of  the  scalp  and  nasopharynx   a.  Submental  lymphnodes   b.  Posterior  cervical  triangle   c.  Anterior  crvical  triangle   d.  Supraclavicular  lymphnode     7.  Nasal  flaring  is  associated  with   a.  Respiratory  distress   b.  Chronic  nasal  obstruction  

15  

  c.  Mouth  breathing   d.  Allergic  rhinitis     8.  A  hole  in  nasal  septum  is  commonly  caused  by   a.  Syphilis   b.  Tuberculosis   c.  Cocaine  abuse   d.  Repeated  trauma  in  picking  off  crusts     9.  Headache  presents  on  aakening   a.  Migraine   b.  Brain  tumor  headache   c.  Meningitis   d.  Subarachnoid  hemorrhage     10.  Sudden  unilateral  painless  visual  loss   a.  Acute  glaucoma   b.  Corneal  ulcer   c.  Uveitis   d.  Retinal  vein  occlusion     Test  II:  Match  the  abnormalities  of  the  lips  with  description  below   a.  Angular  cheilitis   b.  Cold  sore   c.  Chancre  of  syphilis   d.  Carcinoma  of  the  lips   e.  Angioedema        B    11.  Produce  recurrent  and  painful  eruptions  of  the  lips  and  surrounding  skin      A      12.  Maybe  due  to  ill-­‐fitting  dentures      C      13.  Highly  infectious,  firm,  button-­‐like  lesion  that  ulcerates  and  may  become  crusted        D      14.  Fair  skin  and  prolonged  exposure  to  the  sun  are  common  risks  factors      A      15.  It  may  be  due  to  nutritional  deficiency     Test  III:  Match  the  pattern  of  hearing  loss  with  description  below   a.  Conductive  hearing  loss   b.  Sensorineural  hearing  loss        A      16.  One  cause  is  otitis  media      B      17.  In  weber  test,  sound  lateralizes  to  good  ear  

16  

     B      18.  In  Rinne  test,  normal  pattern  prevails      A      19.  Voice  maybe  loud  because  hearing  is  difficult      A      20.  Usual  ageof  onset  childhood  and  adulthood,  up  to  age  40                                                                            

17  

 

18  

HEENT   a.  Conductive  hearing  loss   b.  Sensorineural  hearing  loss     1.

Voice  may  be  loud  because  the  patient  has  trouble  hearing  his  or  her  own  voice        B      

2.

Age  of  onset  is  most  often  in  childhood  and  young  adulthood,  up  to  age  40        A      

3.

Otitis  media        A      

4.

Sound  lateral  to  good  ear  in  Weber  test        B      

5.

AC>BC,  Rinne  test        B        

6.

Basic  landmark  –  cricoid  cartilage  

7.

Lesion  in  the  posterior  2/3  of  the  scalp  and  nasopharynx  –  subscapular  

8.

Retrosternal  goiter  –  venous  engorgement  

9.

Examination  of  the  lymphnode  –  palpation  

10. Ophalmoscopic  examination   a.  The  view  is  limited  to  posterior  structure   b.  Pheripheral  structures  can  be  evaluated  in  the  absence  of  mydriatic  drops   c.  (+)3  or  (+)4  diopters  will  allow  you  to  see  the  anterior  structures  clearly   d.  Red-­‐orange  reflex  is  normally  not  visualize  first   11. A  (-­‐)  lens  is  used  in   a.  Hyperopic  eyes   b.  Aphakic  eyes   c.  Myopic  eye   d.  Astigmatism     12. Firm,  button-­‐like  lesion  –  chancre  of  syphilis   13. Risk  factors  are  fair  skin,  and  prolonged  exposure  to  sun  –  carcinoma  of  the  lip   14. Nutritional  insufficiency  –  angular  cheilitis   15. Ill-­‐fitting  dentures  –  angular  cheilitis   16. Painful  vesicular  lesions  in  angle  of  the  mouth  –  herpes  simplex/cold  sore   17. Unilateral,  painless  visual  loss   a.  Acute  glaucoma   b.  uveitis   c.  retinal  vein  occlusion   d.  all  of  the  above   18. Headache  upon  waking  up  –  migraine   19. Nasal  flaring  associated  to   a.  Respiratory  distress   b.  Chronic  nasal  obstruction   c.  Mouth  breathing  

 

19   d.  Allergic  rhinitis   20. Hole  in  the  basal  septum  most  common  in   a.  Syphilis   b.  Tuberculosis   c.  Cocaine  abuse   d.  Repeated  trauma  in  picking  off  crusts                                                                      

  Test  I:  Choose  the  BEST  answer   1.  In  opthalmoscopic  Examination   a.  The  view  is  limited  to  posterior  structure   b.  Pheripheral  structures  can  be  evaluated  in  the  absence  of  mydriatic  drops   c.  (+)3  or  (+)4  diopters  will  allow  you  to  see  the  anterior  structures  clearly   d.  Red-­‐orange  reflex  is  normally  not  visualize  first     2.  A  (-­‐)  lens  is  used  in   a.  Hyperopic  eyes   b.  Aphakic  eyes   c.  Myopic  eye   d.  Astigmatism     3.  Physical  sign  of  retrosternal  goiter   a.  Venous  engorgement   b.  Tender  thyroid   c.  Thyroid  bruit   d.  Regional  lymphnode  enlargement     4.  Examinatin  of  the  lymphnode  is  primarily  by   a.  Inspection   b.  Palpation   c.  Auscultation   d.  X-­‐ray     5.  Basic  landmark  for  thyroid  gland  examination   a.  Cricoid  cartilage   b.  Thyroid  cartilage   c.  Trachea   d.  Sternocleidomastoid     6.  Primary  lesion  from  posterior  2/3  of  the  scalp  and  nasopharynx   a.  Submental  lymphnodes   b.  Posterior  cervical  triangle   c.  Anterior  crvical  triangle   d.  Supraclavicular  lymphnode          

20  

  7.  Nasal  flaring  is  associated  with   a.  Respiratory  distress   b.  Chronic  nasal  obstruction   c.  Mouth  breathing   d.  Allergic  rhinitis     8.  A  hole  in  nasal  septum  is  commonly  caused  by   a.  Syphilis   b.  Tuberculosis   c.  Cocaine  abuse   d.  Repeated  trauma  in  picking  off  crusts     9.  Headache  presents  on  aakening   a.  Migraine   b.  Brain  tumor  headache   c.  Meningitis   d.  Subarachnoid  hemorrhage     10.  Sudden  unilateral  painless  visual  loss   a.  Acute  glaucoma   b.  Corneal  ulcer   c.  Uveitis   d.  Retinal  vein  occlusion     Test  II:  Match  the  abnormalities  of  the  lips  with  description  below   a.  Angular  cheilitis   b.  Cold  sore   c.  Chancre  of  syphilis   d.  Carcinoma  of  the  lips   e.  Angioedema        B    11.  Produce  recurrent  and  painful  eruptions  of  the  lips  and  surrounding  skin      A      12.  Maybe  due  to  ill-­‐fitting  dentures      C      13.  Highly  infectious,  firm,  button-­‐like  lesion  that  ulcerates  and  may  become  crusted        D      14.  Fair  skin  and  prolonged  exposure  to  the  sun  are  common  risks  factors      A      15.  It  may  be  due  to  nutritional  deficiency          

21  

  Test  III:  Match  the  pattern  of  hearing  loss  with  description  below   a.  Conductive  hearing  loss   b.  Sensorineural  hearing  loss        A      16.  One  cause  is  otitis  media      B      17.  In  weber  test,  sound  lateralizes  to  good  ear      B      18.  In  Rinne  test,  normal  pattern  prevails      A      19.  Voice  maybe  loud  because  hearing  is  difficult      A      20.  Usual  ageof  onset  childhood  and  adulthood,  up  to  age  40                                                                

22  

 

23  

HEENT   a.  Conductive  hearing  loss   b.  Sensorineural  hearing  loss     1.

Voice  may  be  loud  because  the  patient  has  trouble  hearing  his  or  her  own  voice        B      

2.

Age  of  onset  is  most  often  in  childhood  and  young  adulthood,  up  to  age  40        A      

3.

Otitis  media        A      

4.

Sound  lateral  to  good  ear  in  Weber  test        B      

5.

AC>BC,  Rinne  test        B        

6.

Basic  landmark  –  cricoid  cartilage  

7.

Lesion  in  the  posterior  2/3  of  the  scalp  and  nasopharynx  –  subscapular  

8.

Retrosternal  goiter  –  venous  engorgement  

9.

Examination  of  the  lymphnode  –  palpation  

10. Ophalmoscopic  examination   a.  The  view  is  limited  to  posterior  structure   b.  Pheripheral  structures  can  be  evaluated  in  the  absence  of  mydriatic  drops   c.  (+)3  or  (+)4  diopters  will  allow  you  to  see  the  anterior  structures  clearly   d.  Red-­‐orange  reflex  is  normally  not  visualize  first   11. A  (-­‐)  lens  is  used  in   a.  Hyperopic  eyes   b.  Aphakic  eyes   c.  Myopic  eye   d.  Astigmatism     12. Firm,  button-­‐like  lesion  –  chancre  of  syphilis   13. Risk  factors  are  fair  skin,  and  prolonged  exposure  to  sun  –  carcinoma  of  the  lip   14. Nutritional  insufficiency  –  angular  cheilitis   15. Ill-­‐fitting  dentures  –  angular  cheilitis   16. Painful  vesicular  lesions  in  angle  of  the  mouth  –  herpes  simplex/cold  sore   17. Unilateral,  painless  visual  loss   a.  Acute  glaucoma   b.  uveitis   c.  retinal  vein  occlusion   d.  all  of  the  above   18. Headache  upon  waking  up  –  migraine   19. Nasal  flaring  associated  to   a.  Respiratory  distress   b.  Chronic  nasal  obstruction   c.  Mouth  breathing  

 

24   d.  Allergic  rhinitis   20. Hole  in  the  basal  septum  most  common  in   a.  Syphilis   b.  Tuberculosis   c.  Cocaine  abuse   d.  Repeated  trauma  in  picking  off  crusts        

                                                             

  Our  Lady  of  Fatima  University   College  of  Medicine  –  Regular  class   nd 2  semester  2013-­‐2014   Clinical  Medicine   HEENT   Name:  _____________________________________  Section:  ______________  Date:  _________  Score:______     1.  Major  attributes  to  headache                              a.  Location   c.  Chronological  pattern     b.  Quality   2.  Headache  is  episodic  and  tends  to  peak  several  hours.  This  is:     a.  Migraine     c.  brain  tumor     b.  Cluster  headache   d.  meningitis   3.  Nausea  and  vomiting  is  common  in:     a.  Meningtis     c.  Brain  tumor     b.  Tension  headache   d.  Rebound   4.  Cough,  sneezing  ang  changing  patternof  the  head  can  increase  the  pain  from:     a.  tension  headache   c.  Brain  tumor     Meningitis/migraine   d.  cluster  headache   5.  Aging  vision:   Presbyopia   6.  Bilateral  bilateral  painful  eye  maybe  due  to  the  following     a.  Chronic  radiation  exposure   c.  Central  retinal  occlusion     b.  Cholinergic  medication     d.  corneal  ulcer   7.  Horizontal  diplopia     a.  weakness  or  paralysis  of  EOM     c.  lesion  CN  III  or  CN  V     b.  Lesion  CN  III  or  IV       d.  problems  in  cornea   8.  In  conductive  hearing  loss     a.  Have  particular  trouble  understanding  speech     b.  Noisy  environment  makes  hearing  worse     c.  Problem  in  inner  ear     d.  noisy  environment  help   9.  perception  that  the  patient  or  the  environment  is  rotating  or  spinning.  This  is:     a.  Tinnitus     b.  Vertigo     c.  Dizziness     d.  Meniere’s  disease   10.  perceived  sound  that  has  no  external  stimulus:   a.  dizziness   11.  sensation  of  spinning:   Vertigo   12.  Fever,  pharyngeal  exudates,  anterior  lymphadenopathy  without  cough:     a.  viral  pharyngitis     c.  Diptheria     b.  Strep  Pharyngitis     d.  Infectious  Mononucleosis     13.  Hyperthyroidism     a.  Cold  intolerance       c.  Weight  loss     b.  Preference  of  warm  clothing       d.  Decreased  sweating   nd 14.  Cause  of  blindness  in  African  American  and  2  leading  cause  of  blindness  overall?   a.  Cataract     c.  weight  loss     b.  Glaucoma       d.  decreased  sweating   15.  20/40  corrected  is:       a.  The  patient  can  read  the  line  40  with  glasses     b.  Patient  can  read  the  line  without  glasses     c.  Vision  is  normal     d.  Patient  is  presbyopic    

25  

  16.  An  enlarged  blind  spot  occurs  in  condition  affecting  optic  nerve  such  as:     a.  Optic  atrophy     c.  Glaucoma     b.  Cataract     d.  Papilledema   17.  Contraindication  for  mydriatic  drops     a.  Coma       c.  Arcus  senillis     b.  Cataract     d.  pterygium   18.  The  Tug  test  is  painful  in:     a.  Otitis  media     c.  chronic  otitis  externa     b.  Acute  otitis  externa   d.  purulent  otitis  media   19.  Unilateral  sensory  neural  loss     a.  Sound  is  heard  at  the  good  ear     b.  Sound  is  heard  in  the  impaired  ear       c.  Due  to  impact  cerumen     d.  due  to  eardrum  perforation   20.  The  nasal  mucosa  is  pale,  bluis  or  red.  This  suggests:     a.  Viral  rhinitis     c.  allergic  rhinitis     b.  Acute  sinusitis     d.  normal  mucosa   21.  A  triangular  thickening  of  the  bulbar  conjunctiva:    a.  pterygium   22.  Characteristic  of  Optic  atrophy     a.  Visible  optic  vessel   c.  absent  optic  vessel     b.  tiny  optic  vessel   d.  no  vessel  pulsation   23.  Normal  retinal  artery     a.  Arterial  wall  transplant       c.  Narrow  light  reflex     b.  narrow  column   of  blood  vessel     d.  focal  narrowing   24.  Fullness  and  popping  sound  in  the  ear  with  mild  conductive  hearing  loss  and  ear  pain     a.  Acute  otitis  media     c.  with  effusion     b.  Chronic  otitis  externa   25.  Bulging  eardrum     a.  Hearing  loss  is  sensorineural   c.  accentuated     b.  Obscured       d.  changes  in  atmospheric  pressure   26.  The  patient  complains  of  earache  and  hearing  loss.  The  eardrum  are  reddened.  Losses  its  landmark  and   buldges  laterally  towards  the  examiners  eye     a.  Acute  otitis  media  with  purulent  effusion     b.  Chronic  otitis  externa     c.  Chronic  otitis  media     d.  Acute  otitis  externa   27.  The  skin  of  the  ear  canal  is  often  thickened,  red  and  itchy:  Chronic  otitis  externa   28.  Button  like  lesion:   Chancre  syphilis   29.  Benign  condition  that  may  follow  antibiotic  therapy:     a.  Geographic  tongue   c.  smooth  tongue     b.  Fissured  tongue   d.  hairy  tongue   30.  Deficiency  in  riboflavin,  niacin,  folicacid,  B12  and  pyrodoxin     a.  Atrophic     c.  Hairy     b.  Fissured     d.  Geographic   31.  Basic  landmark  of  the  thyroid     a.  Thyroid  cartilage   c.  Trachea     b.  Cricoid  cartilage   d.  Isthmus   32.  Furosemide  –  medication  that  affect  the  hearing   33.  Nose  picking  for  local  cause  of  epistaxis   34.  Rhinitis  medicamentosa  –  excessive  use  of  decongestant   35.  Cause  of  excessive  tearing   36.  Absence  of  red  reflex   37.  Bilateral/unilateral  exopthalmus  

26  

 

27  

38.  Actinic  cheilitis   39.  Diphtheria   40  Canker  sore  –  a  painful  round  or  ovalulcer  that  is  white  or  yellowish  gray  that  is  surrounded  by  halo  reddened   mucosa   41.  Tophi  –  deposits  of  uric  acid  crystals                                                                                                

    1.

2.

3. 4.

5.

6. 7.

8. 9.

10. 11.

12.

13. 14.

15.

28  

Vision  of  20/200  means  that     a. at  20  ft,  the  patient  can  read  print  that  a  person  with  normal  vision  could  read  at  200  ft   b. at  200  ft,  the  patient  can  read  print  that  a  person  with  normal  vision  could  read  at  20  ft.   c. the  larger  the  first  number,  the  worse  the  vision   d. normal  vision   An  image  from  the  upper  nasal  visual  field  strikes  the   a. Upper  temporal  area   c. Lower  temporal  area   b. Lower  nasal  area   d. Upper  nasal  area    Sees  better  when  the  card  is  farther  away   a. Nearsightednesss   c. Presbyopia   b. Myopia   d. hyperopia    Absence  of  red  reflex  indicates   a. Artificial  eye   c.     b. Opacity  of  lens   d.      An  enlarged  blind  spot  occurs  in   a. Grave’s   c. Optic  neuritis   b. Retroorbital  tumor   d. Diabetic  neuropathy    Headache  from  errors  of  refraction  include   a. nearsightedness   c. astigmatism   b. farsightedness   d. myopia   Testing  near  reaction  is  used  in  diagnosis  of   a. Argyll  Robertson  pupil   c. Oculomotor  nerve  paralysis   b. Anisocoria   d. Horner’s  syndrome      Fixed  defects  (scotoma)  are  seen  in   a. Retina   c. Lens   b. Cornea   d. Pupil      Excessive  tearing  from  increased  production  is  due  to   a. Corneal  irritation   c. Entropion   b. Extropion   d. Nasolacrimal  duct  obstruction    Most  important  attribute  of  headache   a. Severity   c. Quality   b. Chronological  pattern   d. location   Hyperthyroidism   a. Intolerance  to  cold   c. Involuntary  weight  loss   b. Preference  for  warm  clothing   d. Decreased  sweating     The  tug  test  is  painful  in   a. Otitis  media   c. Chronic  otitis  externa   b. Acute  otitis  externa   d. Purulent  otitis  media      Bilateral,  painless  change  in  refractory  may  be  due  to   a. Chemical  exposure   c. Steroids   b. Radiation  exposure   d. Diabetes    Family  history  may  be  positive  is   a. Tension   c. Migraine   b. Cluster   d. Medication  obveruse    The  eardrum  itself  is  scarred,  no  landmarks  visible,  often  closes  in  healing  processs   a. Tympanosclerosis   c. Otosclerosis   b. Perforation  of  eardrum   d. Serous  effusion  

  Retinal  Arteries  and  Arteriovenous  Crossing     C   16.  Arteries  may  show  focal  or  generalized  narrowing   B   17.  Opaque  wall  and  no  blood  visible      

   

a.  Copper  wore   b.  Silver  wire  

 

29  

A   18.  Arteries  close  to  the  disc  become  full  and  somewhat  torturous   c.  Retinal  Arteries  in  Hypertension   C   19.  Arterial  wall  is  invisible  A-­‐V  crossing  is  visible       d.  Normal  Retinal  Artery   C   20.  Light  reflex  is  narrow  about  one-­‐fourth  the  diameter  of  blood  column     Pupillary  Abnormalities     D   21.  pupils  that  accommodate  but  do  not  react  to  light       a.  Anisocoria   C   22.  dialted  pupil  is  fixed  to  light  and  near  effort       b.  Adie’s  pupil   B   23.  slow  accommodation  causes  blurred  vision       c.  Oculomotor  nerve  paralysis   B   24.pupil  is  large  regular,  usually  unilateral         d.  Argyll  Robertson  pupil   A   25.causes  include  blunt  trauma  to  the  eyes,  open-­‐angle  glaucoma       Diplopia     A   26.  Images  are  side  by  side         a.  Horizontal  diplopia   A   27.  Caused  by  palsy  of  CN  III  or  IV       b.  Vertical  diplopia   D   28.  Diplopia  in  one  eye  with  the  other  closed       c.  Both     B   29.  Images  are  on  top  of  each  other         d.  None   A   30.  One  kind  of  this  diplopia  is  physiologic    

 

30  

  Hearing  Loss     A   31.  Abnormality  is  usually  visible  except  in  the  otosclerosis       a.  Conductive  hearing  loss   B   32.  In  Weber  test,  room  noise  not  appreciated       b.  Sensorinueral  hearing  loss   B   33.  Voice  is  loud  because  hearing  is  difficult         c.  Both   C   34.  To  estimate  hearing,  test  one  ear  at  a  time       d.  None   B   35.  In  Rinne  test,  the  normal  pattern  prevails     Eardrum     D   36.  Pink,  grayish,  intact           a.  Serous  Effusion   C   37.  Caused  by  acute  purulent  infection  of  the  middle  ear   b.  Perforated  Eardrum   B   38.  Eardrum  itself  is  scarred,  no  landmarks  are  visible   c.  Acute  Otits  Media  with  Purulent  Effusiion   A   39.  Amber  fluidbehind  the  eardrum  is  characteristic.     d.  Normal  Eardrum              Air  bubbles  can  be  seen  with  the  amber  fluid   C   40.  Eardrum  is  bulging  and  redden,  most  landmarks  are  obscured     Clinical  Findings     D   41.  Diptheria           a.  Benign  midline  lump   F   42.  Apthous  ulcer           b.  Nasal  sputum   G   43.  Epulis           c.  Poor  convergence   E   44.  Fever  blister           d.  Pseudomembrane   J   45.  Microaneurysm         e.  Cold  sore   H   46.  Sentinel  node           f.  Canker  sore   B   47.  Transilumination         g.  Pregnancy  tumor   I   48.  Hyperthyroidism         h.  Thoracic  malignancy   K   49.  Hypothyroidism         i.  (+)  lidlag   A   50.  Torus  palatus           j.  Diabetes                 k.  None  of  the  above       1. Major  attributes  to  headache   a. Location   b. Quality   c. Chronologic  pattern   d. Hindi  kya  AOTA  to?  Kasi  tama  lahat   2. Headache  is  episodic  and  tends  to  peak  after  several  hours.   a. Migraine   b. Cluster   c. Brain  tumor   d. Meningitis   3. Nausea  and  vomiting  is  common  in   (MIGRAINE  ang  sagot,  wala  s  choices.  Lahat  ng  choices  walang  assoc.  nausea  and  vomiting)   a. Meningitis   b. Tension   c. Brain  tumor   d. Rebound  

 

31  

  4.

5. 6.

7.

8.

9.

10. 11. 12.

13.

14.

 

Cough,  sneezing,  changing  position  of  the  head  can  increase  the  pain  from:   a. Tension  headache  –  sustained  muscle  tension   b. Migraine  –  noise  and  bright  lights   c. Brain  tumor  (and  sinusitis)   d. Cluster  –  alcohol   Aging  vision  –  PRESBYOPIA   Bilateral  painful  eye   a. Chronic  radiation  exposure   b. Cholinergic  medication  –  bilateral,  painless   c. Central  retinal  artery  occlusion  –  unilateral,  painless   d. Corneal  ulcer-­‐  unilateral,  painful   Horizontal  diplopia   a. Weakness  or  paralysis  of  EOM   b. Lesion  of  CN  III  or  IV   c. Lesion  of  CN  III  or  VI   d. Corneal  problems   Conductive  hearing  loss   a. Have  particular  trouble  understanding  speech  –  sensorineural   b. Noisy  environment  makes  hearing  worse  –  sensorineural   c. Problem  in  inner  ear  –  sensorineural   d. Noisy  environment  may  help   Perception  that  the  patient  or  the  environment  is  spinning\   a. Tinnitus  –  perceived  sound  without  external  stimulus   b. Vertigo   c. Dizziness   d. Meniere’s  disease  –  tinnitus  +  hearing  loss  +  vertigo   Perceived  sound  without  external  stimulus  –  TINNITUS   Sensation  of  spinning  –  VERTIGO  (hindi  kaya  dizziness  to?)   Fever,  pharyngeal  exudates,  anterior  lymphadenopathy,  no  cough   a. Viral  pharyngitis   b. Strep  pharyngitis   c. Diphtheria   d. Infectious  mononucleosis   Hyperthyroidism   a. Cold  intolerance  –  hypo   b. Preference  of  warm  clothing  –  hypo   c. Weight  loss   d. Decrease  sweating  –  hypo   nd Leading  cause  of  blindness  in  African  American  and  2  leading  cause  of  blindness  overall   a. Cataract   b. Glaucoma   c. Macular  degeneration   d. Retinal  detachment  

 

32  

  15. 20/40  corrected  is   a. Patient  can  read  line  40  with  glasses   b. Patient  can  read  the  line  without  glasses   c. Vision  is  normal   d. Presbyopia   16. An  enlarged  blind  spot  occurs  in  conditions  effecting  the  optic  nerve  such  as:  (glaucoma,  optic  neuritis,   papilledema)   a. Optic  atrophy   b. Cataract   c. Glaucoma   d. Papilledema   17. Contraindications  for  mydriatic  drops  (head  injury,  coma,  suspicion  of  narrow  angle  glaucoma)   a. Coma   b. Cataract   c. Arcus  senilis   d. Pterygium   18. Tug  test  is  painful  in     a. Otitis  media   b. Acute  otitis  externa   c. Chronic  otitis  externa   d. Purulent  otitis  media   19. Unilateral  sensorineural  loss   a. Sound  is  heard  in  good  ear   b. Sound  is  heard  in  impaired  ear  –  unilateral  conductive   c. Impacted  cerumen   d. Eardrum  perforation   20. The  nasal  mucosa  is  pale,  bluish,  or  red   a. Viral  rhinitis   b. Acute  sinusitis   c. Allergic  rhinitis   d. Normal  mucosa   21. Triangular  thickening  of  bulbar  conjunctiva  –  PTERYGIUM   22. Characteristic  of  optic  atrophy   a. Visible  optic  vessel   b. Tiny  optic  vessel   c. Absent  optic  vessel   d. No  vascular  pulsation   23. Normal  retinal  artery   a. Arterial  wall  is  transparent   b. Narrow  column  of  blood  vessel  –  HPN   c. Narrow  light  reflex   d. Focal  narrowing  –  HPN   24. Fullness  and  popping  sound  in  the  ear  with  mild  conductive  hearing  loss  and  ear  pain  –  SEROUS  EFFUSION   25. Bulging  eardrum  (Parang  wala  tamang  sagot.  Dapat  acute  otitis  media  with  purulent  effusion)   a. Hearing  loss  is  sensorineural  –  conductive  

 

33  

26.

27. 28. 29.

30.

31.

b. Obscured   c. Accentuated   d. Changes  in  atmospheric  pressure  –  serous  effusion   The  patient  complains  of  earache  and  hearing  loss.  The  eardrums  are  reddened,  loses  its  landmark,  and   bulges  lateral  towards  examiner’s  eye.   a. acute  otitis  media  with  purulent  effusion   b. chronic  otitis  media  externa   c. chronic  otitis  media   d. acute  otitis  externa   skin  of  the  ear  canal  is  often  thickened,  red  and  itchy  –  CHRONIC  OTITIS  EXTERNA   Button  like  lesion  –  CHANCRE  IN  SYPHILLIS   Benign  condition  that  may  follow  antibiotic  therapy       a. Geographic  tongue  –  dorsum  shows  scattered  smooth  areas  denuded  of  papillae   b. Fissured  tongue  –  increasing  age   c. Smooth  tongue  /  atrophic  glossitis  –  deficiency  of  riboflavin,  niacin,  folic  acid,  vitB12,  pyridoxine,  iron   or  treatment  with  chemotherapy   d. Hairy  tongue  –  HIV,  AIDS   Deficiency  of  riboflavin,  niacin,  folic  acid,  B12,  pyridoxine   a. Atrophic  glossitis   b. Fissured   c. Hairy   d. Geographic   Basic  landmark  of  thyroid  cartilage   a. Thyroid  cartilage   b. Cricoid  cartilage   c. Trachea   d. Isthmus                                          

 

34  

EXTREMITIES   Clinical  Medicine  EXTREMITIES     1. Test  for  nerve  irritation..     a. Trendelenberg  –  HIP  DISLOCATION  (PAINLESS  LIMPING)   b. Faber  Test   c. Thomas  Test  –  HUGH  OWEN  THOMAS  SIGN  (TEST  THIGH  AND  HIP  JOINT)  LUMBAR   PELVIC  DISEASE,  HIP  JOINT  DISEASE   d. None     THOMAS  SIGN-­‐  TEST  FLEXION  (FLATTENING  OF  LUMBAR  CURVE  IS  OBSERVE)     2. Type  of  primary  joint  articulation  that  allows  free  movement..     a. Fibrous   b. Synovial   c. Cartilagenous   d. All     3. Manifestation  /  Cause  of  extreme  dorsiflexion  foot     a. Ruptured  Achilles  tendon  –  absence  plantar  flex  (simmond’s  test)   b. Soleus  tear  –  malformation  leg  severe  pain  and  tenderness   c. Gastrocnemius  strain   d. All     4. Site  of  muscle  of  the  rotator  except     a. Infraspinatus   b. Supraspinatus   c. Teres  major   d. None     5. ROM  of  elbow   Ans.  Flexion,  extension,  supination,  pronation     6. Anterior  knee  cyst  –  INFLAMMATION  OF  THE  BURSA     a. Housemaid  –  PREPATELLAR  BURSITIS   b. Clergyman  –  INFRAPATELLAR  BURSITIS   c. Baker’s  cyst  –  MORRANT  BAKER  POPLITEAL  ARTERY  ENTRAPMENT   d. A  &b     BAKER   -­‐  COMPLICATION  OF  RA     7. Osteoarthritis     a.Heberden’s  nodes   b.  Bouchard’s  nodes   c.  asymmetric..  

LORDOSIS,  

  d.  all     8. Palmar  erythema  except     a.portal  HPN   b.aplastic  anemia   c.  pregnancy   d.valvular  heart  dse       9. Wrist  ROM     Ans.  Flexion,  extension,  abduction,  adduction     10. True  regarding  neck  ROM     a.flexion  and  extensionat  the  skull  and  C1  –  C3-­‐C7   b.rotation  at  c1  and  c2  –  ATLANTO-­‐AXIAL   c.a&b   d.  none     11. Visual  examination  from  behind..  landmarks  except     a.spinous  process  of  T1  -­‐  PALPATION   b.iliac  crest   C.POSTERIOR  SUPERIOR  ILIAC  SPINE   d.  dimple  of  venus     12. Test  for  Lumbar  lordosis   ANS.  THOMAS  TEST     13. Examination  of  motor  function     a.look  for  tremor   b.look  for  coordinated  movement   c.muscle  size   d.all     14. Pronated  hand  dropped  from  wrist   a. Carpal  tunnel  syndrome   b. Ulnar  nerve  palsy   c. Median  nerve  palsy  (MEDIAN  =  RADIAL)   d. None     15. Lateral  deviation  from  midline   ANS.  GENU  VALGUM           16. Rheumatoid  arthritis     A.MIP  &POP  INVOLVEMENT  

35  

 

36  

B.HAYGARTH’S  NODES   c.Bouchard’s  nodes  -­‐  OA   d.all   e.a&b     17. Unequal  shoulder  length     A.SCOLIOSIS   B.SPRENGEL’S  DEFORMITY   c.a&b   d.none     18. Unequal  leg  length  except   Ans.  Kyphosis     19. Long  narrow  nails  except     a.hypopotuitarism   b.eunochoidism   c.  cretinism  -­‐  SQUARE   d.none     20. Lordosis  except   Ans.  A.  …convexity       NOTE:  Kindly  Recheck  all  the  answers!     God  bless!                  

 

37   A. Matching  type  

 

   

E  1.  Drumstick  fingers   C  2.  Spider  fingers          3.  Subungal  haemorrhage   A  4.  Graenlen's  test   H  5.  Lesague's  test   E  6.  Hypertrophic  osteoarthropathy   G  7.  Trendelenburg's  sign          8.  Tinel'  sign   K  9.  Infrapatellar  bursitis        10.  Genu  recurvatum   M  11.  Acromegaly   F  12.  Azure  half-­‐moon   D  13.  Hypothenar     -­‐-­‐-­‐-­‐-­‐   -­‐-­‐-­‐-­‐-­‐  

    B.  MULTIPLE  CHOICE     16.  Thumb  in  Filipino  is:   A.  Hinlalaki   B.  Palasingsingan     17.  Hinlalato  is:   A.  Thumb   B.  Index  finger     18.  Severe  carpal  tunnel  syndrome  is  managed  by:   A.  NSAIDs   B.  Physical  therapy     19.  Not  a  malposture  of  the  hand:   A.  Claw  hand   B.  Ape  hand     20.  Most  common  hypothesis  of  clubbing:   A.  Unknown  mechanism   B.  Hypoxia     21.  Lateral  bending  of  the  spine:   A.  Atlanto-­‐axial  joint   B.  Midcervical  vertebra     22.  Eggshell  nails:   A.  Vitamin  A  deficiency   B.  Hypochromic  anemia      

A.  Passive  hyperextension   B.  Radial  nerve  injury   C.  Arachnodactyly   D.  Ulnar  nerve  injury   E.  Clubbing   F.  SBE   G.  Hip  dislocation   H.  Straight  leg  test   I.  Carpal  tunnel  syndrome   J.  Housemaid's  knee   K.  Clergyman's  knee   L.  Small  patella   M.  Square  &  round  nail  plate   N.  -­‐-­‐-­‐-­‐-­‐   O.  -­‐-­‐-­‐-­‐-­‐   P.  -­‐-­‐-­‐-­‐-­‐  

C.  Hinlalato   D.  None  

C.  Middle  finger   D.  None   C.  Complete  rest  of  the  wrist   D.  Surgical  decompression  

C.  Wrist  drop   D.  Polydactyly   C.  Parrot  beak  nails   D.  (?)  

C.  Atlanto-­‐occipital   D.  C3-­‐C7   C.  Thyrotoxicosis   D.  Bronchiectasis

                                       

  23.  Brittle  nails:   A.  Onycholysis   C.  Onychocrytosis   B.  Onychorrhexis   D.  Onychocryptosis     24.  Ram's  horn  nails:   A.  Onycholysis   C.  Onychocrytosis   B.  Onychorrhexis   D.  Onychocryphosis     25.  Fixation  of  the  2nd  toe  in  flexion:   A.  Hallus  valgus   C.  Hammer  toe   B.  Hallus  rigidus   D.  Callus     26.  Point  of  reference  of  the  fingers:   A.  Wrist  joint   C.  Ulnar  nerve   B.  Middle  finger   D.  Radial  nerve     27.  Rheumatoid  arthritis  is  mostly  seen  in   A.  Fingers   C.  Elbow   B.  Knees   D.  Hip  joint     28.  Pain  referred  to  the  shoulder:   A.  Pneumonia   C.  Winged  scapula   B.  Supraspinatus  tendinitis   D.  All     29.  Excessive  transverse  growth  of  the  nail  plate  causing  lateral  edge  to  lacerate:   A.  Onychauxis   C.  Ram's  horn  nails   B.  Onychocryptosis   D.  Onychogryphosis     30.  One  of  the  following  regarding  the  examination  of  the  spine  is  incorrect:   A.  From  the  side  inspect  the  spinal  profile   B.  From  behind  inspect  the  lateral  curve   C.  Percuss  the  spine  with  a  neurological  hammer   D.  Use  your  thumb  to  palpate  for  the  para  vertebral  area     31.  One  of  the  following  is  a  cause  of  kyphosis  in  postmenopausal  women:   A.  Osteoporosis   C.  Osteus  deformans   B.  Faulty  posture   D.  Ankylosing  spondylitis     32.  When  the  legs  deviate  towards  the  midline  and  the  knees  farther  apart,  it  is  called:     A.  Genu  varum   C.  Genu  recurvatum   B.  Genu  valgum   D.  None  of  the  above     33.  Presence  of  blood  in  the  joint  cavity:   A.  Hemarthrosis   C.  Hemoptysis   B.  Hematemesis   D.  Hematochezia       34.  The  length  of  the  lower  extremities  can  be  measured  by:   A.  ASIS  to  the  tip  of  the  medial  malleolus  with  the  tape  crossing  the  patella   B.  ASIS  to  the  tip  of  lateral  malleolus   C.  From  iliac  crest  to  Achilles  tendon   D.  From  iliac  crest  to  medial  malleolus  

38  

    35.  When  doing  the  lumbar  puncture,  iliac  crest  is  at:   A.  4th  lumbar  vertebrae   C.  T12-­‐  L1   B.  L1-­‐L2   D.  S4       36.  Extreme  dorsiflexion  of  the  foot   A.  Ruptured  Achilles  tendon   C.  Soleus  tear   B.  Fracture  of  the  tibial  shaft   D.  Fracture  of  the  fibular  shaft     37.  Pressure  diverticulum  of  the  synovial  sac  protruding  thru  the  joint  capsule  of  the  knee   A.  Baker's  cyst   C.  Pyarthrosis   B.  Popliteal  abscess   D.  Prepatellar  bursitis     38.  Swelling  in  the  popliteal  fossa   A.  Baker's  cyst   C.  Prepatellar  bursitis   B.  Popliteal  abscess   D.  Infrapatellar  bursitis     39.  Which  of  the  following  is  the  most  common  cause  of  painful  swelling  in  young  males?   A.  Paget's  disease   B.  Ankylosing  spondylitis   C.  Pott's  disease   D.  Osteoporosi

39  

      UPPER  EXT   1.  Hands  (ROM  –  ABDUCTION,  ADDUCTION,  EXTENSION,  FLEXION)   Condition  impaired  ROM  =  FAT   1. Fibrosis  of  palmar  fascia  (DUPUYTREN’S  CONTRACTURE)   2. Arthritis   3. Tenosynovitis  (inflammation  of  tendon  sheaths)     Size  hands   LONG   ACROMEGALY   AFTER  EPIPHYSIS  CLOSE  (ADULT)   EXCESSIVE  PRODUCTION  GH   NOT  SYMMETRICAL   GIGANTISM   BEFORE  EPIPHYSIS  CLOSE  (CHILD)   PROPORTIONATE  AND  SYMMETRIC   ND NOTE:  BOTH  ARE  2  TO  TUMOR  +  BY  AN  EXCESS  OF  SOMATOTROPHIC  HORMONE  FROM  EOSINOPHILIC  ADENOMA  OF   ANTERIOR  PIT  GLAND     LOND  AND  SLENDER  HANDS   *SPIDER  FINGERS   SMALL,  THICK  HANDS   CRETINISM  (CHILD)   DWARF  PX   MYXEDEMA  (OLD)   ABSOLUTE  DEF  OF  TH   HANDS  SHORT,  THICK,  FAT   RADIUS  MAY  BE  SHORTENED   MONGOLISM   CHROMOSOMAL  ABN   HANDS  SHORT,  THICK,  THUMB  DIVERGES  FROM   NEARER  THE  WRIST  THAN  NORMAL   LITTLE  FINGER  IS  CURVED  (RADIAL  WARD)     MALPOSTURE  (ABNORMALITY  IN  POSTURE)  =  CAW   CLAW  HAND   BUSMA   BRACHIAL  PLEXUS   ULNAR  NERVE  INJURIES   SYRINGOMYELIN   MUSCULAR  ATROPHIES   ACUTE  POLIOMYELITIS   APE  HAND   PAS   PROGRESSIVE  MUSCULAR  ATROPHY   AMYOTROPHIC  LATERAL  SCLEROSIS   SYRINGOMYELIN   WRIST  DROP   RPP   RADIAL  NERVE  INJURIES   POLIOMYELITIS   POISONING  (LEAD,  ARSENIC,  ALCOHOL)   2.  PALM   ABNORMALITIES     CAROTENODERMA   HEPATIC  DISEASE   MYXEDEME   THENAR  ATROPHY   MEDIAN  NERVE   HYPOTHENAR  ATROPHY   ULNAR  NERVE   HANSEN’S     THICK  AND  CORD  LIKE  

40  

 

PALMAR  ERYTHEMA  

  3.  FINGERS   MALFORMATION   POLYDACTYL  (SUPERNUMENARY  FINGER)  

SYNDACTYL  (WEBBED  FINGERS)   HEBERDEN’S  NODES  (OSTEOARTHRITIS)  

HAYGARTHS  NODES  (RHEUMATOID  ARTHRITIS)  

  5.  DIGITS   INFECTION   PARONYCHIA  

FELON  

  6.  FINGERNAILS   MALFORMATION     ONYCHORREXIS   -­‐ BRITTLE  NAIL  PLATE   -­‐ BORDER  FRAYED  AND  TORN     CLUBBING  W/O  PERIOSTOSIS   -­‐ CONVEX   ND -­‐ 2  HYPOXIA   -­‐ PARROT-­‐HIPPOCRATIC-­‐SERPENT-­‐DRUMSTICK   ABSENCE  OF  NAILS   -­‐ CONGENITAL   BITTEN  NAILS   -­‐ IRREGULAR   -­‐ SHORT  NAILS   SQUARE  AND  ROUND  

41   HYPOTHENAR  =  FLAT  AND  ATROPHIED   +NUMBNESS   LLV   LIVER  CIRRHOSIS   LATE  STAGE  PREGNANCY   VULVAR  HEART  DISEASE  

LAURENCE  –  BIEDL  SYNDROME   ASSOCIATED  WITH:   1. JUVENILE  OBESITY   2. RETINAL  DEGENERATION   3. GENITAL  HYPOPLASIA   4. MENTAL  RETARDATION   CONGENITAL/HEREDITARY   OSTEOARTHRITIS   PAINLESS  –  DIJ   HARD   OLD  WOMEN   MEN  (DUE  TO  TRAUMA,  SINGLE  JOINT)   INFLAMMATORY   MIDDLE  AND  PROXIMAL   FUSIFORM   PAINFUL   JOINT  CAPSULE  THICKENED  

SWOLLEN   REDDENED   PAINFUL   ABSCESS  TERMINAL  PULP   ND 2  BACTERIAL  INFECTION   ONSET  SWELL  AND  DULL  PAIN  

MALNUTRITION   IRON  DEFICIENCY   THYROTOXICOSIS   CALCIUM  DEFICIENCY   TB   COPD   BROCHIECTASIS   ICHTHYOSIS   TRAUMA   PERSONALITY  DISORDER   NEUROLOGIC   CAS   ACROMGELY  

  LONG  AND  NARROW  

SPOON     -­‐ CONCAVE   -­‐ KOILONYCHOSIS  

EGGSHELL   -­‐ CONCAVE   ONYCHAUXIS   -­‐ HYPERTHOPHY   -­‐ NAILS  ON  TOP  OF  ANOTHER   -­‐ IRREGULAR  DISCOLORED   -­‐ UNKNOWN  CAUSE   RED  HAFT  MOON   AZURE  BLUE  HAFT  MOON   ONYCHOLYSIS   -­‐  SEPRATION  OF  NAIL  FROM  NAILBED  

42   CRETINISM   HEM   HYPOPITIUTARISM   EUNOCHOIDISM   MARFAN’S  SYNDROME   HIS   HYPOCHROMIC  ANEMIA   IRON  DEFICIENCY   RARE:  RLS   -­‐ RHEUMATIC   -­‐ LICHEN  PLANUS   -­‐ SYPHILIS   VIT  A  DEFICIENCY   ND

2  TO  FUNAL  INFECTION   WASHER  WOMEN  

CARDIAC   LANULA  RED   WILSON   LANULA  BLUE   ND 2  TO  FUNAL  INFECTION  AND  TRAUMA   +FOUL  ODOR   -­‐ THYROTOXICOSIS   -­‐ ECZEMA   -­‐ PSORIASIS   -­‐ MYENTERIC  DISEASE   BACTERIAL  ENDOCARDITIS   TRICHINOSIS  

SUB  UNGCAL  HEMORRHAGE   -­‐ SPLINTER  HEMMORAGE   -­‐ LINEAR  RED   -­‐ EMBOLIC   BEUS’S  LINE   ACUTE  SEVERE  ILLNESS   -­‐  TRANSVERSE  DEPRESSION     7.  WRIST  –  ROM  DORSIFLEX,  PALMARFLEXION,  ULNAR  AND  RADIAL  DEVIATION   CONDITIONS   A.CHRONIC  ARTHRITIS     –  PAINFUL  AND  FUSIFORM  ENLARGMENT   B.  NON-­‐SUPPURATIVE  TENOSYNOVITIS     –  PAINFUL  SWELLING  ANATOMIC  SNUFFBOX     -­‐  (SAUSAGE  LIKE  SWELLING)   -­‐  TENDON  SHEATHS  (EXTERNAL  POLLICIS  BREVIS  AND  ABDUCTOR  POLLICIS  LONGUS)   -­‐  +CREPITUS   -­‐  DUE  TRAUMA  AND  INFLAMMATION  (GOUT  AND  GONOCCOCAL)   C.  CARPAL  TUNNER  SYNDROME     -­‐  COMPRESSION,  NEUROPATHY  MEDIAN  NERVE     -­‐  (+)  TPN  (TINGLING,  PAIN,  NUMBNESS)     -­‐  FLEXOR  CARPI  RADIALIS  AND  FLEXOR  CARPI  ULNARIS     -­‐  PHYSICAL  SIGN       ATROPHY       HYPERSTHESIAS       PROGESSIVE  WEAKNESS  AND  AWKWARDNESS     NOTE:  

 

43   1. 2.

TINEL’S  SIGN  –  TINGLING  PALMARIS  LONGUS   PHALEN  SIGN  –  HYPEREXTENSION  BOTH  HANDS  FOR  3  MINS  (PRAYER  SIGN)  

  8.  FOREARM   -­‐  MOST  SUSCEPTIBLE  FRACTURE   -­‐  VOLAR  MASS  FORM  BY  FLEXORS   -­‐  SMITH’S  FRACTURE       9.  ELBOW   -­‐  SWELLING  MORE  COMMON  EXTENSOR  SURFACE   -­‐  RHEUMATOID  NODULE  –  FOUND  OLECRANON  BURSAE  AND  ULNAR  REGION   NOTE:     HUMERO-­‐ULNAR  =  EXTENSION  AND  FLEXION   HUMERO-­‐RADIAL  =  PRONATION  AND  SUPINATION         CUBITUS  VALGUS  =  OUTWARD  (ANGLE  LESS  THAN  170)     CUBITUS  VARUS  =  INWARD         OLECRANON  BURSITIS  =  STUDENT  MINER’S   ND   ARTHRITIS  ELBOW  =  TENNIS  2  TENDONITIS     10.  SHOULDER     WINGED  SCAPULA   -­‐ SPRENGELS  SCAPULA   -­‐ PARALYSIS  LONG  THORACIC  NERVE   -­‐ CONGENITAL   -­‐ SOMETIMES  ASSOCIATED  WITH  SHORT  WEBBLED  NECK     11.  SPINE     NODDING  AND  LIFTING  =  ATLANTO-­‐OCCIPITAL   FLEXION  AND  EXTENSION  =  C3  AND  C7   LATERAL  BENDING  =  MIDCERVICAL   ROTATION  =  ATLANTO-­‐AXIAL  (C1-­‐C2)     CONDITIONS:   PANCOAST   -­‐  SUPERIOR  PULMO  SULCUS  SYNDROME   -­‐  TUMOR  PULMO  APEX,  UPPER  MEDIASTINUM  AND   SUPERIOR  THORACIC  APERTURE   +NECK  PAIN   PARESIS  OR  ATROPHY  OF  ARM  MUSCLE  MAY  OCCUR   HORNER   UNI  MIOSIS,  PTOSIS  OF  EYELIDS   -­‐SWEAT   TUMOR  LUNG  APEX  OR  NECK   CERVICAL  SPONDYLOSIS   DEGENERATION  OF  NUCLEUS  PULPOSUS   CERVICAL  OSTEOARTHRITIS   WHISPLASH   RUPTURE  OF  LIGAMENTUM  NUCHAE   SUDDEN  FORCEFUL  HYPEREXTENSION  OF  NECK  WITH   HYPERFLEX  RECOIL     POST  TRAUMA   HYPERREFLEXION  OF  NECK  –  C5   PARTIAL  DISLOCATION  FROM  HYPEREXT  –  UNABLE  TO  NOD   FRACTURE  ATLAS  –  SEVER  OCCIPITAL  HEADACHE  

    THORACOLUMBAR  CONDITIONS   1. WHIPLASH  CERVICAL  –  GRADUAL  NUMBNESS  AND  TINGLING  SENSATION  THAT  GOES  DOWN  TO  THE  HAND   2. KYPHOSIS  –  FORWARD  DEFORMITY  OF  THORACIC  (HUNCHBACK)   3. SMOOTH  CURVE   4. ANGULAR  CURVE  -­‐  +GIBBUS   5. LORDOSIS  –  POSTERIOR  CONCAVITY  OF  LUMBAR  SPINE,  DEEP  FURROW,  POT  BELLY   6. SCOLIOSIS  –  FEMALE  WALK  SEXY,  CONGENITAL,  PARALYSIS  BACK  OR  ABDOMINAL  MUSCLE     LOWER  EXT     1.  HIP  JOINT  AND  THIGH     TEST  FOR  HIP   LESAQUE’S  TEST   STRAIGHT  LEG  RAISING   SCIATIC  NERVE   TEST  THE  RANGE  OF  HIP  FLEXION   PATRICS   LATERAL  ROTATION  OF  THE  HIP   GRAELEN’S   PASSIVE  HYPEREXTENSION   ACTIVE  HYPEREXTENSION   ONLY  TEST  IN  PRONE     2.  KNEE   CONDITIONS   GENU  VARUM   BOW  LEG   LEGS  DEVIATE  TOWARD  MIDLINE   1. RICKETS   2. OSTEITIS  DEORMANS   3. COWBOY  AND  JOCKEY   GENU  VALGUM   KNOCKED  KNEE   LATERAL  DEVIATION  OUTWARD   GENU  RECURVATUM   KNEES  FIXED  IN  HYPEREXTENSION  WITH  LITTLE  ABILITY   TO  FLEX   ABSENSE  PATELLA     1.  Which  of  the  following  disease  manifest  a  big  hand,  big  bones  and  muscles?   a.  Gigantism  b.  Acromegaly  c.  Polydactyl  d.  Marfan  syndrome     2.  What  is  the  action  of  axiohumeral  muscle  group?     a.  Internal  rotation  of  shoulder     b.  External  rotation  of  shoulder     c.  Shrug  shoulder  at  the  back     d.  All  of  the  above     3.  What  is  the  action  of  axioscapular  muscles?  (same  choices)     4.  This  disease  is  cause  of  adenoma  in  pituitary  gland  related  to  hypothyroidism?     a.  Cretinism  b.  Ehler  danlos  syndrome  c.  mongolism     5.  The  following  are  changes  seen  in  carpal  tunnel  syndrome?     a.  Thenar  atrophy  b.  Hypothenar  atrophy  c.  Ape  hand  d.  All  of  the  above     6.  Which  of  the  following  has  a  smooth  curve  of  spine?     a.  Cancer  b.  Infectious  spondylitis  c.  Ankylosis  spondylitis  d.  Pott’s  disease    

44  

 

45  

7.  Which  of  the  following  has  an  angular  curve  of  spine?     a.  Ostetitis  deformans  b.  Ankylosis  spondylitis  c.  Senile  osteoporosis  d.  Infectious  spondylitis   8.  Manifestation  of  Achilles  tendon  damage  .  Positive  simmond  test  .  Body  weight  is  applied  to  base  of  the  foot  because  of   plantar  flexion  .  Because  of  hyper  extension  and  dorsiflexion  .  A  and  b  only     9.  This  is  associated  with  eunochoidism     a.  Eggshell  nail  b.  Spoon  nails  c.  Long  narrow  nails  d.  Square  and  round  nails     10.  Sub  ungal  haemorrhage  is  the  cause  of  the  following  disease  except?     a.  Cardiac  disease  b.  SBE  c.  Trichinosis  d.  None  of  the  above     11.  This  line  is  associated  with  acute  severe  illnesses  –  Beau’s  Line  This  is  associated  with  renal  disease  something?haha!   forgot  exact  na  sakit  a.  Red  half  moon  b.  Blue  half  moon  c.  Onychauxis  d.  None  of  the  above     12.  Test  for  lateral  rotation     a.  Graenlen  b.  Stretching  of  legs  upward  c.  Patrick  d.  Owen   Test  for  hip  flexion  (same  choices)   13.  Type  3  pitting  edema  is?     a.  2mm  b.  4mm  c.  6mm  d.  8mm     14.  This  is  a  congenital  disease  associated  with  damage  to  long  thoracic  nerve?  –  Winged  scapula     15.  Location  of  tumor  in  pancoast  syndrome     a.  Apex  b.  Mediastinum  c.  Base  d.  A  and  b  e.  All  of  the  above     16.  Responsible  for  movement  of  neck  from  left  to  right     a.  C3-­‐c7  b.  Atlanto  occipital  c.  Axial  d.  Midcervical  vertebrae     17.  Scoliosis  can  be?     a.  Paralysis  of  back  muscle  b.  Congenital  c.  1  leg  is  paralyzed  d.  A  and  b  e.  All  of  the  above     18.  Genu  recurvatum     a.  Fixed  hyperextension  b.  Fixed  flexed  c.  Congenital  d.  A  and  c     19.  Morrant  baker  cyst  is  associated  with?  –  Popliteal  artery  syndrome     20.  Genu  Valgum?     a.  Lateral  deviation  of  leg  b.  Bow  leg  c.  Cause  of  rickets  d.  .  ?     21.  Genu  Varus     a.  Can  be  cause  of  occupational  b.  Paget’s  disease  c.  Can  be  cause  by  rickets  d.  All  of  the  above     22.  Associated  with  Marfan  Syndrome     a.  Long  slender  nails  b.  Arachnodactyl  c.  Claw  hand  d.  A  and  b  e.  All  of  the  above     23.  Cause  of  claw  hand  a.  Brachial  plexus  injury  b.  Syringomyelia  c.  Muscle  dystrophy  d.  All  of  the  above     24.  Nodes  found  in  DIP  is  associated  with?     a.  Osteoarthirits  b.  Systemic  rheumatic  arthritis  c.  Gout  arthritis  d.  Infectious  arthritis     25.  Combing  of  hair  is  associated  with?     a.  Internal  rotation  and  abduction  b.  External  rotation  and  abduction  c.  Internal  rotation  and  adduction  d.  External   rotation  and  adduction     26.  Movement  of  the  right  arm?    

  a.  Counter  clockwise  on  pronation  and  clockwise  on  supination  b.  Counterclockwise  on  supination  and  clockwise  on   pronation  c.  Both  are  correct  d.  None  of  the  above   27.  What  should  you  assess  when  looking  at  the  spine  sideward?   a.  Height  of  shoulder   b.  Curvature   c.  Location  of  ASIS   d.  All  of  the  above     28.  What  should  you  assess  if  you  are  looking  at  the  back  of  patient?   a.  Height  of  shoulder   b.  Dimples?   c.  ASIS   d.  All  of  the  above     30.  If  a  person  cannot  raise  greater  than  10  degrees  in  elbow  joint?   a.  Dislocation   b.  Fracture   c.  Damage  of  supraspinatus  tendon   d.  All  of  the  above     31.  Heloma  Durum  is?   a.  Hard  corn   b.  Callus   c.  Soft  corn   d.  Stiffened  toe     32.  If  there  will  be  a  fracture  in  humerus,  what  should  you  check?   a.  Brachial  artery   b.  Radial  artery   c.  Brachioradial  artery   d.  ?   Medicine  Extremities   1. Dorsiflexion  –  Soleus  Tear   2. Housemaid’s  –  anterior  knee  cyst   3. To  test  range  of  hip  flexion,  nerve  irritation  and  lumbar  herniation  –  straight  leg  test   4. Test  done  to  detect  excessive  lumbar  convexity  /  lordosis  –  Hugh  Owen  Thomas  sign   5. Yellow  discoloration  in  palms  imparted  to  the  skin  by  carotene  –  carotenemia   6. Inspection  of  vertebral  column  viewed  laterally  –  AOTA,  cervical,  thoracic  ,  lumbar  curve   7. True  about  lordosis  except:  -­‐  posterior  convexity  of  lumbar  sign                    Posterior  concavity  of  lumbar  sign                    Deep  furrow  between  paraspinous  muscle                    Pot  belly     8. Test  for  hip  dislocation  –  trendelenberg’s  sign   9. What  is  the  joint  that  allows  free  movement  like  knee  and  shoulder  joint  –  fibrous            Synovial            Cartilaginous   10. SITS  muscle  of  rotator  cuff  muscle  except:  -­‐  supraspinatus              Infraspinatus              Teres  minor              Subscapularis  Muscle              None   11. ROM  of  ELBOW  –  extension  ,  pronation  flexion,  supination  

46  

 

47   12. Drop  arm  definition  (bates)  -­‐  if  patient  cannot  hold  arm  fully  abducted  at  shoulder  level,  possible  rotator  cuff   tear.   13. This  can  be  seen  in  osteoarthritis    -­‐  heberdens     14. Bouchards  -­‐  rheumatoid  arthritis              Asymmetric  deformitiesof  the  hands  and  wrists              All   15. Range  of  motion  for  the  wrist:  flexion,  extension,  abduction,  adduction   16. Hypertrophy  of  nailplates  caused  by  chronic  fungal  infection  –  onychauxis   17. Long  narrow  nail  plates  except:  cretinism   18. Pronated  wrist  drop  –  radial  nerve  palsy     19. Rotation  of  neck  –  rotation  at  c1  and  c2   20. Difference  in  shoulder  length  is  due  to  :  scoliosis   Sprengels  shoulder             A  and  b             NONE   21. View  person  from  spine  is  for  :  cervical  convexity                            Thoracic  convexity                                  Lumbar  convexity                                  None   22. Matching  type:  hugh  owen   23. Stretch  sciatic  muscle  –  straight  leg  raising   24. Lateral  rotation  grip  –  patricks  test   25. Passive  hyperextension  –  graelens  test   26. Eversion  –  talipes  valgus   27. Dorsiflexion,  defect  on  neural  arch  –  talipes  calcaneus   28. Genu  varum  occur  in  –  rickettes                                                          Pagets                              Both   29. Length  of  lower  extremities  –  from  ASIS  to  medial  malleolus   30. Thumping  of  posterior  spine  –  volar  surface  of  wrist   31. Spoon  nail  –  iron  deficiency   32. Test  for  acromioclavicular  joint  –  cross  over  test   33. Bowleg  –  genu  varum   34. Bitten  nails  common  to  personality  disorder   35. Knocked  knee  –  genu  valgum   36. Onychauxis  –  chronic  fungal  infection   37. Subungal  hemorrhage  –  splinter  hemorrhage  linear  red  hemorrhage  in  nailbed   38. Square  and  round  nail  plates  –  cretinism     Unequal  Leg  Length  can  be  seen  -­‐  Scoliosis,  Hip  Fracture,  Poliomyelitis    except:  Kyphosis    

             

 

48  

  EXTREMITIES   1. Lumbar  concavity:      false  regarding  lordosis(dapat  lumbar  concavity)   2. Soleus  tear:   extreme  dorsiflexion   3. Flexion  and  extension  occur  primarily  between  the  skull  and  C1,  the  atlas;  rotation  at  C1-­‐C2;  both   4. Sits  muscles:     supraspinatus,infraspinatus,teres  minor  and  subscapularis   5. In  osteoarthritis,  Heberden’s  nodes  at  DIP  joints,  Bouchard’s  nodes  at  the  PIP  joint.      In  rheumatoid  arthritis,  symmetric  deformity  in  the  PIP,MCP  and  wrist  joints  with  ulnar  deviation.   6. Wrist  movement:     flex,extend,  abduct,  adduct   7. Wrist  drop:     radial  nerve   8. Unequal  shoulder  heights  seen  in  scoliosis;:   Sprengel’s  deformity  of  the  scapula(from  the  attachment  of  an  extra  bone  or  band  between  the  upper  scapula   and  C7);  in  “winging”  of  the  scapula  (from  loss  of  innervations  of  the  serratus  anterior  muscle  by  the  long   thoracic  nerve);  and  in  contralateral  weakness  of  the  trapezius   9. Anterior  knee  cyst:     prepatellar(housemaid’s  knee)  and  infrapatellar(clergyman’s  knee)   10. Genu  varum:     legs  deviate  towards  the  midline     11. Long  narrow  nails:     all  except  cretinism(included:  Marfan’s,  eunochoidism,  hypopituitarism)   12. Palmar  erythema:     except  hemolytic  anemia(included:  pregnancy,  portal  HPN,  vulvar  heart  dse)     13. Motion  of  elbow:     supination,  pronation,  flexion,  extension   14. Thomas  sign:     test  for  lumbar  lordosis   15. Lesaque’s  test   16. Synovial:    freely  movable(knee,  shoulder)   17. Scoliosis,  kyphosis,  hip  fracture     18. You  may  wish  to  percuss  the  spine  for  tenderness  by  thumping,  but  not  too  roughly  with  your  fist   19. Patrick’s  test   20. Dorsiflexion:     either  ankle  motion  or  talipes  calcaneus   21. Housemaid’s:   anterior  knee  cyst   22. To  test  range  of  hip  flexion,  nerve  irritation,  and  lumbar  herniation:     straight  leg  test   23. Test  done  to  detect  excessive  lumbar  convexity/lordosis:     Hugh  owen  Thomas  sign   24. Yellow  discoloration  in  palms  imparted  to  the  skin  by  carotene:     carotenemia   25. Inspection  of  vertebral  column  viewed  laterally:    

 

49   26.

27. 28. 29. 30. 31. 32. 33. 34. 35. 36. 37. 38. 39. 40. 41. 42. 43. 44. 45. 46. 47. 48. 49. 50.

AOTA,  cervical,  thoracic,  lumbar  curve   True  about  lordosis  except:     posterior  convexity  of  lumbar  sign,  post  concavity  of  lumbar  sign,  deep  furrow  between  paraspinous  muscle,   pot  belly   Test  for  hip  dislocation:    trendelenberg’s  sign   What  is  the  joint  that  allows  free  movt  like  knee  and  shoulder  joint:     fibrous,  synovial,  cartilaginous   SITS  muscle  of  the  rotator  cuff  muscle:     supraspinatus,  infraspinatus,  teres  minor,  none   ROM  of  elbow:       extension,  pronation,  flexion,  supination   Drop  arm  definition(BATES)   This  can  be  seen  in  osteoarthritis:     heberden’s,  bouchard’s,  asymmetric  deformities  of  the  hands  and  wrists,  all   Range  of  motion  of  the  wrist:     flexion,  extension,  abduction,  adduction   Hypertrophy  of  the  nail  plates  caused  by  fungal  infection:   onychauxis   Pronated  wrist  drop:     radial    nerve  palsy   Rotation  of  the  neck:     rotation  at  c1  and  c2   Difference  in  shoulder  length  is  due  to:     scoliosis,  sprengel’s  shoulder,  a  and  b,  none   Long  narrow  nail  plates  except:  Cretinism   View  person  for  spine  is  for:  cervical  convexity,  thoracic  convexity,  lumbar  convexity,  none   Matching  type:  hugh  owen   Stretch  sciatic  nerve:     straight  leg  raising   Lateral  roattion  grip:     Patrick’s  test   Passive  hyperextension:    graenlen’s  test   Eversion:     talipes  valgus   Dorsiflexion,  defect  on  neural  arch:     talipes  calcaneus   Genu  varum  occurs  in:     rickettes,  paget’s,    both   Length  of  the  lower  extremities:     from  ASIS  to  medial  malleolus   Thumping  of  posterior  spine:     volar  surface  of  wrist   Spoon  nail:    iron  deficiency   Test  for  acromioclavicular  joint:     cross  over  test  

 

50   51. Bowleg:     genu  varum   52. Bitten  nails  common  to:     personality  disorder   53. Knocked  knee:    genu  valgum   54. Onychauxis:     chronic  fungal  infection   55. Subungal  hemorrhage:     splinter  hemorrhage  linear  red  hemorrhage   56. Square  and  round  nailplates:     cretinism   57. Dorsiflexion:     either  ankle  motion  or  talipes  calcaneus   58. Test  done  to  detect  excessive  lumbar  convexity/lordosis:   hugh  owen  Thomas  sign   59. True  about  lordosis  except:     posterior  convexity  of  lumbar  sign   60. Test  for  hip  dislocation:     trendelenberg’s  sign   61. Joint  that  allows  free  movement  like  knee  and  shoulder  joint:     synovial  joint   62. ROM  of  elbow:    extension,  pronation,  flexion,  supnation   63. Rotaion  of  neck:     rotation  at  c1  and  c2   64. Difference  in  shoulder  length:     scloiosis,  sprengel’s  shoulder   65. Lateral  rotation:     patrick’s  test   66. Passive  hyperextension:     graenlen’s  test   67. Eversion:     talipes  valgus   68. Stretch  sciatic  nerve:     straight  leg  test                          

 

51   1.

One  of  the  following  regarding  examination  of  spine  is  incorrect   a. From  the  side  inspect  the  spinal  profile   b. From  behind  inspect  lateral  curves   c. Percuss  the  spine  with  a  neurological  hammer   d. Inspect  and  palpate  paravertebral  muscles   2. Conditions  that  impair  range  of  motion  of  fingers  and  wrist  are:   a. Arthritis   b. Tenosynovitis   c. Fibrous  in  the  palmar  fascia   d. AOTA   e. A  &  B   3. All  of  the  following  are  associated  with  Marfan’s  syndrome  Except:   a. Thumb  sign   b. Hyperextensible  joints   c. Hypertrophic  osteoarthropathy   d. Elongated  long  bones   4. The  following  statements  about  carpal  tunnel  syndrome  are  correct  Except:   a. There  is  atrophy  of  the  radial  half  of  thenar  eminence   b. Pain  especially  at  night  is  experience   c. Weakness  and  loss  of  finer  movements  is  not  common  on  this  disease   d. Trauma  from  excessive  flexion  of  the  wrist  is  common  cause   5. Which  of  the  following  statements  is  associated  with  clawhand:   a. Claw  is  formed  by  hyperextension  of  the  interphalangeal  joint   b. The  condition  maybe  secondary  to  Rheumatoid  arthritis   c. Brachial  plexus  injury  is  one  of  the  possible  cause   d. AOTA   6. Ruptured  Achilles  tendon  is  manifested  by:   a. pain  at  the  heel   b. inability  to  plantar  flex  the  foot   c. inability  to  dorsiflex  the  foot   d. shortening  of  the  tendon   7. Pain  on  elevation  of  the  arm  60-­‐120  degrees  is  due  to:   a. Chronic  tendinitis   b. Partial  rupture  of  of  supraspinatous  tendon   c. Dislocation   d. Complete  fracture   8. One  of  the  following  is  a  cause  of  kyphosis  in  post  menopausal   a. Osteoporosis   b. Faulty  posture   c. Osteitis  deformans   d. Ankylosing  spondylitis   9. Gibbus  deformity  maybe  secondary  to:   a. Pagets  dse   b. Hypercalcemia   c. Metastatic  carcinoma   d. Cervical  spondylosis   10. Internal  rotation  of  the  shoulder  joint  is  tested  by:   a. Raising  both  arms  to  a  vertical  position  

 

52  

11.

12.

13.

14.

15.

b. Putting  both  hands  behind  the  small  ___  of  the  back   c. Putting  both  hands  behind  the  neck   d. NOTA   st Among  the  tests  of  the  hip  joint  the  most  gentle  test  that  should  be  done  1  is:   a. Anvil  test   b. Active  hyperextension   c. Graenlen’s  test   d. Rotation  of  the  thigh     Deviation  of  the  legs  toward  the  midline  ,  leads  to  condition  called:   a. Genu  valgum   b. Bowlegs   c. Knock  knee   d. Genu  recurvatum   Popliteal  artery  entrapment  syndrome  maybe  secondary  to:   a. Prepatellar  bursitis   b. Popliteal  abscess   c. Clergyman’s  knee   d. NOTA   The  landmarks  of  the  shoulder  joint  are:   a. Coracoid  process,  medial  epicondyle  and  greater  tuberosity   b. Acromion,  coracoid  process,  greater  tuberosity  of  humerus   c. Greater  tuberosity,  lateral  epicondyle,  clavicle   d. NOTA   Excessive  transverse  growth  of  the  nail  plate  causing  the  lateral  edge  to  lacerate   a. Onychauxis   b. Onychocryptosis   c. Ram’s  horn  nail   d. onychogryphosis        

                               

 

53   1. 2. 3. 4. 5. 6. 7. 8. 9. 10. 11. 12. 13. 14. 15. A. B. C. D. E. F. G. H. I. J. K. L. M. N. O. P. Q.

                             

  Rheumatoid  Arthritis   Acromegaly   Mongolism   Palmar  Erythema   Phalen’s  Sign/  Tinel’s  Sign   Trendelenberg  test/  Anvil  test   Lesaque’s  sign   Hugh  Owen  Thomas  Sign   Patrick’s  test   Winged  Scapula   Talipes  Equinus   Talipes  Varus   Eggshell  nails   Carotenoderma   Thumb  sign/  Wrist  Sign     Pituitary  tumor   Straight  Leg  Raising   PIPJ   Lateral  Rotation  of  hip   Toe  wear  of  shoes   Short  fat  hand   Yellow  vegetables   Hip  dislocation   Flexure  contracture  of  hip  joint   Liver  Cirrhosis   Arachnodactyly   Sprengels  deformity  Median  Nerve   Median  nerve   Lateral  wear  of  shoes   Active  hyperextension   Little  finger  is  curved   Vitamin  A  deficiency  

Answer Key C A P J M H B I D L E N Q G K

 

54  

      1. 2. 3. 4. 5. 6. 7. 8. 9. 10. 11. 12. 13. 14. 15. A. B. C. D. E. F. G. H. I. J. K. L. M. N. O.                                

Rheumatoid  Arrthritis/  Haygarth’s  Node   Acromicria   Mongolism   Palmar  Erythema     Tinel’s  Sign   Simmond’s  test   Lesaques  sign   Hugh  Owen  Thomas  test   Graenlen’s  test   Azure  Half  moon   Whiplash  Injusry   Nodding  and  lifting  the  head   Fracture  of  humeral  back   Winged  scapula     Olecranon  Bursitis     Little  finger  is  curved   PIPJ   Passive  hyperextension  shoulder  pain  small  hand   Flexion  contracture,  hip  joint   Straight  leg  raising   Ruptured  ligamentum  nuchae   Median  nerve   Achilles  tendon   Hepatolenticular  regeneration   Cirrhosis   Short  thick  hands   Atlanto  occipital  joint   Paralysis  long  thoracic  nerve   Miner’s  elbow   Atlanto  axial  joint  

Answer Key B E A L I J G F C K H N D O P

 

55  

EXTREMITIES   1.  Popliteal  artery  entrapment  syndrome  maybe  due  to:   A.Housemaid’s  cyst      B.  Infrapatellar  cyst   C.  Popliteal  abcess   D.  Large  Baker’s  cyst   2.  Splinter  hemorrhage  in  nailbeds  is  suggestive  of:   A..SubAcute  bacterial  endocarditis          B.  Anemia       C.  Hepatolenticular  degeneration     D.  Iron  deficiency   3.  Congenital  small  patella     A.Genu  Varum   B.  Genu  Recurvatum     C.  Genu  Valgus   D.  Degenerative  osteoarthritis   4.  The  thumb  is  held  in  extension  by  its  inability  to  flex     A.Ape  hand   B.Claw  hand   C.Wrist  drop   D.  Benediction  Hand   5.  In  performing  the  ROM  of  the  spine  the  examiner  should  assist  the  elderly  patient  by     A.Holding  one  hand  of  the  patient   B.Asking  him  to  be  careful  C.Asking  him  to  hold  Hand  Rails     D.Stabilizing  the  pelvis  by  holding  the  pelvis  of  the  patient  while  patient  performs  ROM   6.  Lesion  of  the  Ulnar  Nerve  leads  to:     A.Wrist  drop   B.  Claw  hand   C.  Hypothenar  atrophy   D.  Palmar  Erythema   7.    As  the  examiner  positions  himself  behind  the  patient,  he  does  the  ff:     A.Inspect  for  lateral  curves   B.  Locate  the  dimples  of  venus   C.  Take  note  of  the  height  of  shoulders  and  iliac  crest     D.Inspect  cervical,  thoracic  and  lumbar  curves   8.  When  looking  for  rheumatic  nodules  the  most  likely  area  to  look  is:     A.Medial  Epicondyle   B.  Extensor  surface  near  olecranon  process   C.Knee  Joint   D.  Lateral  Epicondyle   9.  The  paravertebral  muscles  are  evaluated  by:     A.Inspection  for  fasiculations   B.  Ascultation  for  crepitus  C.  Palpate  for  tenderness  and  spasm   D.All  are  correct   10.  Motions  of  the  forearm:     A.Pronation  &  Supination   B.  Flexion  &  Extension     C.  Both     D.  Neither   11.  The  popular  theory  as  to  the  cause  of  clubbing  of  the  finger  is/are:     A.Floating  nail   B.-­‐20  degress  angulation  of  the  nail     C.hypozia   D.  All   12.  Haygarth’s  Node  except     A.Distal  Interphalangeal  joint      B.  Rheumatoid  arthritis      C.Prodromal  Interphalengeal  joint    D.  Metacarphophalengeal  gout   13.  A  casue  of  kyphosis  in  young  males  due  to  painful  disease  of  the  spine  is     A.Paget’s  Disease  B.  Osteoporosis   C.  Faulty  posture  D.Ankylosis  spondylitis   14.  Legs  are  outwardly  deviated:     A.Genu  recurvatum   B.  Genu  varus   C.  Genu  valgum   D.  Osteoarthritis  Knee   15.  Nodding  and  lifting  the  head  involves:     A.Midcervical  Vertebra   B.  C3-­‐C7  C.  Atlantooccipital  joint   D.  Atlantoaxial  joint   16.  Conditions  that  impair    range  of  motion  of  the  hands  and  wrists  because  of  scarring     A.Arthritis   B.Dupuytren’s  Contracture   C.  Tendenitis   D.  ALL   17.  Carpal  Tunnel  Syndrome:   st nd rd th   A.Numbers  if  the  Small  &  Ring  fingers  B.  Numbness  of  the  1 ,  2  ,3  &  4  fingers  C.  Wrist  drop  D.ALL   18.  Ruptures  ligamentum  nuchae     A.C5  fracture   B.  Cervical  Spondylosis   C.  C1  fracture     D.  whiplash  cervical  injury   19.  A  line  drawn  across  the  iliac  spine  approximates  the   nd th th st   A.2  lumbar  vertebra   B.4  lumbar  vertebra   C.7  thoracic  vertebra   D.1  lumbar  Vertebra   20.  Absence  of  plantar  flexion     A.Tibial  Fracture     B.Ruptured  tendon  od  Achilles   C.  Ruptured  Baker’s  cyst   D.  Soleus  tear                          

 

56  

CHEST,  LUNGS,  BOOBS     Medicine  I   nd Midterms  2  Semester  (February,  2013)     1. Acute  onset  but  chronic  progressive.   -­‐ SMOKING   2. Which  is  likely  the  mechanism  of  cough  among  patients  with  heart  failure?   -­‐ DECREASED  LUNG  COMPLIANCE   3. Which  is  likely  the  mechanism  of  cough  among  patients  with  heart  failure?   a. Pressure  in  the  airway   b. Decreased  lung  compliance   c. Inflammation  of  the  airway   d. Thermal  change   4. 50  y.o.  patient  male  with  >  5  months  cough.  He  cough  when  he  lays  down  wearing  tight  clothes.  It  is  productive  with   watery  whitish  sputum.   a. Congestive  heart  failure   b. Gastroesophageal  regurgitation   c. Bronchiectasis   d. Intestinal  fibrosis   5. 45  year  old  male  has  cough  for  3  months  now  and  claims  to  clear  his  throat  every  morning   a. Smoking   b. Post  Nasal  Drip   8. A  62  years  old  female  was  found  unconscious  and  at  the  ER  she  was  noted  with  very  fast  and  deep  breathing  with   fruity  breath.  Her  pattern  of  breathing  is:   a. Biot’s  respiration   b. Tachypnea   c. Kassmaul’s  respiration   d. Hyperpnea   9. 17  yo  female  consulted  at  the  ER  because  of  high  grade  fever  w/o  other  symptoms.  She  is  observed  w/  abnormal   respiration.  Her  abnormal  respiration  expected  to  be:   a. Tachypnea   b. Platypnea   c. Hyperpnea   d. Kussmaul’s   10.  An  IV  drug  user  presents  to  ER  due  to  hemoptysis  and  DOB  but  denies  any  fever  and  cough.  What  is  the  most   common  cause  of  hemoptysis?   a. PTB   b. Lung  Abscess   c. Pneumonia   d. Vasculitis   12. Kussmaul’s  respiration  is  heard  among  patient  with:   a. Meningitis   b. Ascites   c. CVD   d. Renal  failure   13. 38  y/o  female,  rushed  to  ER  because  of  severe  retrosternal  pain,  relieved  by  nitroglycerine;  pain  occurs  when  cold   substance  ingested.   a. Angina   b. Trachitis   c. Esophagitis   d. Esophageal  spasm   14. Acute  onset  of  Difficulty  of  Breathing  precipitated  by  valsalva  maneuver   a. Pulmonary  edema   b. Rupture  of  aortic  aneurism  

 

15. 16. 17. 18.

19. 20. 21. 22. 24.

26.

27.

29. 31.

33.

34.

   

57   c. Pneumothorax   d. Pulmonary  emboli   Most  common  cause  of  hemoptysis   -­‐ Bronchiectasis   Patient  with  heart  failure   -­‐ Orthopnea   Dry  cough  and  unilateral  effusion   -­‐ Trepopnea   Pneumococoniosis  is  due  to  exposure  to:   a. Silica   b. Coal   c. Asbestos   d. Silver   Bagasse   -­‐ Sugar  cane   Primary  malignancy   -­‐ Silica   Psittacosis   -­‐ Parrot   Lagging  of  one  side  of  the  chest  is  best  identified  by:   -­‐ Palpation   Where  is  the  landmark  for  the  posterior  rib?   a. C7   b. T1   c. Superior  scapular  line   d. Inferior  scapular  line   Referenced  used  in  identifying  posterior  ribs   a. C7   b. T1   c. Superior  scapular  line   d. Inferior  scapular  line   Tactile  fremitus  is  usually  decreased  with:   a. Atelactasis   b. Emphysema   c. Bronchitis   d. Pneumonia   Stridor   -­‐ Epiglottis   Normal  breath  sound  heard  in  intrascapular  area   a. Vesicular   b. Bronchovesicular   c. Bronchial   d. Tracheal   Sounds  of  secreations  produce  on  bronchi?   a. Stridor   b. Rhonchi   c. Rales   d. Wheezes   Bilateral  hyperresonance   a. Tension  pneumothorax   b. Pneumomediastinum   c. Chronic  bronchitis   d. Emphysema  

  35. Findings  on  the  exam  of  the  anterior  chest  would  mostly  involved  which  of  the  following:   a. Upper  lobe   b. Middle  lobe   c. Lower  lobe   d. A  &  B   36. Shorter  I:E  ratio   a. Metabolic  acidosis   b. Bronchitis   c. Pleurisy   d. Fever   37. Describe  the  type  of  breathing  for  patient  with  heart  failure:   a. Cheyne-­‐stokes   b. Biot   c. Kusmail   d. Apneustic   38. Fremitus  is  decreased   a. Pneumonia   b. Obesity   c. Pleural  effusion   d. Atelectasis   40. Early  inspiratory  crackles  is  heard  at:   a. Interstitial  lung  disease   b. Pneumonia   c. CHF   d. Chronic  bronchitis   41. Breast  extends  from?   nd th -­‐ 2  to  6  ICS   42. Most  frequently  palpable  lymph  node  in  Breast  Ca:   -­‐ Central  Lymph  Node   43. Mastitis   -­‐ Lactation   44. Dividing  into  quadrants  is  toL   a. Clinical  finding   b. Staging   46. Seen  in  acute  onset  and  chronic  progressive  DOB:   a. Bronchiectasis   b. Asthma   c. Pneumonia   d. Anemia   47. Orange  peel  in  breast  cancer:   a. Protrusion  of  mass   b. Lymphatic  abscess   c. Venous  abscess   d. Increased  blood  flow       Q:  Mediastinal  mass  cough,  described  as  signs  of  respiratory  distress,  except:     Q:  Diaphragmatic  excursion  except:   a. Atelectasis   b. Pneumonia   c. Pneumothorax   d. Pleural  effusion     Q:  A  35  y/o  consulted  for  fear  of  breast  cancer.  Her  sister  died  of  breast  cancer  at  23  y/o.  What  is  the  risk?  

58  

 

59   a. b. c. d.

>40   2.1-­‐4   1.1-­‐2   None  

  Q:  Suggestive  of  barky  cough     Q:  Brassy  cough   A:  Emphysema     Q:  Brassy   A:  Trachea     Q:  Barky   A:  Glottis     Q:  Pink  puffer   A:  Emphysema     Q:  Mastitis   A:  Lactation     Q:  Man  can’t  keep  up  with  friend  at  same  age   A:  Grade  2     Q:  Occupational  asthma  common  among  workers  exposed  to  which  of  the  following  dust:   a. Asbestos   b. Coal   c. Cotton  dust   d. Silicon     Q:  Pneumocosis   A:  Coal     Q:  Heart  Failure   A:  Orthopnea     Q:  Apex  above  clavicle   A:  2-­‐4cm     Q:  COPD   A:  10x     Q:  Lung  cancer  in  women   A:  13x       Q:  Basilan   A:  Paragoniasis     Q:  Rusty  color  of  sputum   A:  S.  pneumonia     Q:  Pursed  lip   A:  Emphysema  

    Q:  Crescendo=decrescendo  with  apnea   A:  Cheyne  strokes     Q:  Afferent  limb   A:  IX     Q:  Decreased  or  low  fremitus   A:  Thick  chest  wall     Q:  Soft  and  low   A:  Vesicular     Q:  Early  inspiratory  crackles   A:  Chronic  bronchitis     Q:  Most  common  palpable  lymph  node  of  breast  cancer   A:  Central  Node     Q:  High  (4x)  breast  density   A:  Highest  to  relative  risk  for  breast  cancer     Q:  Modifiable  risk  for  breast  cancer   A:  Obesity     Q:  Chronic  cough   A:  2  months     Q:  Cough  complication   A:  Pneumothorax     Q:  IE  ration  is  decreased  in  fever   A:  Chronic  bronchitis     Q:  Chest  pain  associated  with  chest  tenderness  pathology  of   A:  Chest  wall     Q:  Pursed  lip  breathing  is  manifested  by  patients  with:   a. Bronchiectasis   b. Asthma   c. Emphysema   d. Pneumonia     Q:  Pink  puffer  also  known  as   a. Asthma   b. Emphysema   c. Chronic  bronchitis     Q:  Below  are  hereditary  diseases  except   a. HPN   b. Leprosy   c. Heart  Disease   d. DM      

60  

 

61  

  BREAST     1. Intraductal  Papilloma    >  SPONTANEOUS  UNILATERAL  BLODDY  DISHARGE  FROM      ONE/TWO  WARANTS  FURTHER  EVALUATION   2. 1.1–  2.0     >  OBESITY,  LATE  AGE  ETC.   3. Surface  of  Areola     >  SMALL,  ROUNDED  ELEVATION  FORMED  BY  SEBACCEOUS  GLANDS      ,  SWEAT  GLANDS  &  ACCESSORY  AREOLAR   GLANDS   4. Tail  of  Spence     >  EXTENDS  ACROSS  THE  ANTERIOR  AXILLARY  FOLD   5. Female  Breast     ND TH >  CLAVICLE,  2  RIB  DOWN  TO  THE  6  RIB  AND  FROM  THE  STERNUM  ACROSS  TO  THE  MIDAXILLARY  LINE       Recall  –  Clinical  Medicine  “Chest  and  Lungs”  (Dec.  17,  2012)     1.  During  cough,  the  speed  of  airflow  in  the  airway  is  as  fast  as  the  speed  of:     a.  Light           c.  Airplane     b.  Sound         d.  Lightning     2.  What  is  etiology  of  cough  in  pleural  effusion?     a.  Inflammation         c.  Decrease  pulmonary  compliance     b.  Mechanical  irritation       d.  Chemical/thermal     3.  18  y.o.  female  with  acute  onset  of  cough  associated  with  runny  nose,  sore  eyes,  and  chest  discomfort.  What  condition?     a.  Pneumonia         c.  Mycoplasm  pneumonia     b.  Acute  bronchitis       d.  Tracheobronchitis     4.  Chronic  cough,  sputum  purulent,  often  copious  and  foul-­‐smelling     a.  Bronchiectasis       c.  Obstructive  pneumonia     b.  Chronic  bronchitis       d.  PTB     5.  Cause  of  brassy  cough     a.  Glottis         c.  Bronchiole     b.  Trachea         d.  Lung  parenchyma     6.  Patient  with  which  of  the  following  lung  problem  would  present  with  sudden  onset  of  difficulty  of  breathing  preceded   by  pleuritic  chest  pain?     a.  Pulmonary  edema       c.  Pulmonary  emboli     b.  Pneumothorax       d.  Pulmonary  hemorrhage     7.  Sudden  onset  of  difficulty  of  breathing  precipitated  by  exposure  to  grass  pollen     a.  Asthma         c.  Pneumonia     b.  COPD         d.  Hypersensitive  Pneumonitis     8.  Alpha  I  antitrypsin  increase  risk:     a.  Asthma         c.  Chronic  bronchitis     b.  Cystic  fibrosis         d.  Emphysema     9.  49  y.o.  DB,  he  has  DOB  for  3  years.  Lately,  he  can’t  keep  up  with  his  friends  of  the  same  age.  What  is  the  grade  or   American  Thoracic  Dyspnea  Scale?     a.  Grade  1         c.  Grade  3     b.  Grade  2         d.  Grade  4    

 

62  

10.  Considered  as  the  most  common  cause  of  hemoptysis?     a.  Pneumonia         c.  Bronchiectasis  (not  sure)     b.  PTB           d.  Lung  CA     11.  Lung  parenchyma,  common  cause  of  hemoptysis?     a.  Bronchiectasis         c.  Bronchogenic  CA  (not  sure)     b.  Good  pasteur’s  syndrome     d.  Cystic  fibrosis     12.  Squeezing  retrosternal  pain  felt  after  ingestion  of  cold  liquid  and  relieved  by  nitroglycerin  is  felt  in?     a.  Heart           c.  Chest  Wall     b.  GIT           d.       13.  40  y.o.  female  with  rheumatoid  arthritis  of  more  than  15  years  now  and  taking  methotrexate.  Started  to  have  cough   and  difficulty  of  breathing  2  years  ago.  What  respiratory  disease  is  the  most  likely  the  cause?     a.  Non-­‐cardiogenic  pulmonary  edema   c.  Pulmonary  vasculitis     b.  Pulmonary  emboli       d.  Interstitial  infiltrative  disease     14.  Which  of  the  following  occupational  inorganic  substances  will  cause  pneumoconiosis?     a.  Bagasse         c.  Coal     b.  Asbestos         d.  Silica     15.  Exposure  to  which  of  the  following  dusts  increases  the  risk  of  lung  cancer?     a.  Bagasse         c.  Coal     b.  Tin  oxide         d.  Silica     16.  Cigarette  smoking  increases  risk  of  COPD  mortality  by:     a.  2.3x           c.  10x     b.  4x           d.  13x     17.  Smoking  increases  lung  cancer  mortality  in  men  by:     a.  4x           c.  20x     b.  6x           d.  23x     18.  Alcoholics  have  higher  risk  of  getting  pneumonia  and  they  are  more  prone  to  develop:     a.  Aspiration  pneumonia       c.  Pneumonia  due  to  tularemia     b.  Psittacosis         d.  Pneumocystic  pneumonia     19.To  decrease  tension  pneumothorax  where  will  you  insert  the  big  bore  needle:   nd th   a.  2  ICS  MCL  (Mid  Clavicular  Line)   c.  6  ICS  AAL  (Anterior  Axillary  Line)   th th   b.  4  ICS  MAL  (Mid  Axillary  Line)   d.  8  ICS  MAL     20.  Which  of  the  following  vertebrae  has  the  most  prominent  spinous  process     a.  C6           c.  T1     b.  C7           d.     21.  Needle  insertion  of  thoracocentesis   nd th   a.  2  ICS         c.  6  ICS   th th   b.  4  ICS         d.  8  ICS     22.  The  apex  of  the  lungs  is  extended  above  the  inner  1/3  of  the  clavicle  by  as  much  as:     a.  1-­‐3cm           c.  3-­‐5cm     b.  2-­‐4cm         d.  4-­‐6cm     23.  Diaphragmatic  excursion  from  resting  to  full  inspiration  is  at:     a.  1-­‐3cm           c.  5-­‐6cm  

 

63  

  b.  3-­‐5cm           d.  6-­‐7cm     24.  Bifurcation  of  trachea  at:   nd   a.  Manubrium         c.  2  ICS   rd   b.  Sternal  Angle         d.  3  Rib     25.  Pursed  lip  breathing     a.  Emphysema         c.  Asthma     b.           d.     26.  Blue  bloater     a.  Chronic  bronchitis       c.  Bronchiectasis     b.  Emphysema         d.       27.  Ribs  angle  is  usually  more  horizontal  compared  to  normal  individual  if  patient  have  which  of  the  following  problem:     a.  Pneumonia         c.  Pneumothorax     b.  COPD         d.  Mild  Asthma     28.  Normal  angle  of  the  rib:     a.  35  degrees         c.  45  degrees     b.  40  degrees         d.  50  degrees     29.  Narrowing  of  ICS  may  cause  shift  to  the  chest  to  the  one  side:     a.  Massive  effusion       c.  Disc  atelectasis     b.  Severe  pneumonia       d.  Obstruction  of  main  bronchus       30.  Unilateral  widening  and  even  bulging  of  ICS  is  suggestive  of:     a.  Massive  effusion       c.  Large  lung  mass     b.  Severe  pneumonia       d.  Pneumothorax     31.  IE  ratio  in  emphysematous  patient:     a.  1:2           c.  1:4     b.  1:3  (not  sure)         d.  1:5     32.  Which  among  the  following  abnormal  respiratory  rhythm  is  seen  in  the  lesion  of  the  PONS?     a.  Cheyne  stroke         c.  Kussmaul     b.  Biot’s           d.  Apneustic     33.  Diabetic  Ketoacidosis:     a.  Cheyne  stroke         c.  Kussmaul     b.  Biot’s           d.  Apneustic     34.  Type  of  respiration  associated  with  morphine  overdose:     a.  Cheyne  stroke         c.  Kussmaul     b.  Biot’s           d.  Apneustic     35.  Clubbing  of  fingers  are  seen  in  this  pathologic,  except:     a.  Bronchiectasis         c.  Mesothelioma  (not  sure)     b.  COPD         d.  Pulmonary  fibrosis     36.     37.  Which  of  the  following  breath  sounds  have  intermediate  pitch  and  loudness?     a.  Tracheal         c.  Broncho-­‐vesicular     b.  Bronchial         d.  Vesicular  

    38.  Which  of  the  following  breath  sounds  has  loud  and  high  pitched  sound?     a.  Tracheal         c.  Broncho-­‐vesicular     b.  Bronchial         d.  Vesicular     39.     40.  Mid  inspiratory  &  expiratory  crackles  usually  heard  in  patients  with:     a.  Bronchiectasis     41.  Normally  breast  tissue  extend  from  the  sternum  to:     a.  AAL           c.  PAL     b.  MAL           d.  Between  AAL  &  MAL     42.  Rounded  elevations  seen    in  the  areola  is  caused  by  the  following  except:     a.  Sebacceous  glands       c.  Accessory  glands     b.  Sweat  glands         d.  Fat  globule     43.  Fibroadenoma  of  the  breast  is  a  common  cause  of  breast  mass  among  female  aging:     a.  15-­‐20  y.o.         c.  50-­‐65  y.o.     b.  25-­‐50  y.o.         d.  >65  y.o.     44.  40  y.o.  nullipara,  married,  no  family  history  of  breast  CA,  what  is  the  risk  factor?     a.  >4           c.  1.1-­‐2.0  (not  sure)     b.  1-­‐4           d.     45.  Which  of  the  following  can  decrease  the  risk  of  breast  CA?     a.  Use  of  contraceptive       c.  Breast  feeding     b.  Late  pregnancy         d.  Post  menopausal     46.  MRI  of  the  breast  is  recommended  for  the  following  except:     a.  Older  woman         c.  High  risk  group     b.  Newly  diagnosed  breast  CA     d.  Women  with  dense  breast     47.  Galactorrhea  may  be  seen  in  patient  with     a.  Herpes  zoster         c.  Ovarian  CA     b.  Pituitary  adenoma       d.     48.  Benign  condition  of  breast  may  cause  bloody  breast  discharge     a.  Adenofibrosarcoma       c.  Fibroadenoma     b.  Intraductal  papilloma       d.  Neurosarcoma     49.  Breast  malignancy  is  most  likely  to  have  if  breast  tumor  is  about:     a.  2cm           c.  4cm     b.  3cm           d.  5cm  (not  sure)     50.  Breast  malignancy  is  highly  considered  if  the  patient  have       a.  Smooth  border         c.  Palpable  mass     b.  Wall  delineated       d.  Immobile                

64  

 

65  

CARDIOLOGY     Areas  of  Auscultation   nd a. 2  ICS  LSB   nd b. 2  ICS  RSB   c. Erb’s  Point   th d. 4  ICS  LSB   th e. 5  ICS  LMCL   nd

36.  Pulmonary  Stenosis  –  2  ICS  LSB   th 37.  Mitral  Stenosis  –  5  ICS  LMCL   nd 38.  Opening  Snap  –  2  ICS  LSB   th 39.  Mitral  valve  prolapse  –  5  ICS  LMCL   nd 40.  PDA  –  2  ICS  LSB   nd 41.  Aortic  stenosis  –  2  ICS  RSB   th 42.  S3  &  S4  –  5  ICS  LMCL   th 43.  Mitral  regurgitation  –  5  ICS  LMCL   nd 44.  Splitting  of  S2  –  2  ICS  LSB   th 45.  Tricuspid  regurgitation  –  4  ICS  LSB                 Clinical  Medicine:  CVS  exam   1  to  5  choices:   a.  concentric     d.  RVH   b.  eccentric     e.  LVH   c.  LV  dilatation     1.  Not  palpable  in  PE   2.  Forceful  apical  beat  but  not  displaced   3.  Markedly  displaced  at  the  left  axillary  line,  hardly  palpable   th th 4.  Displaced  to  axillary  line  5  or  6  ICS  anterior  axillary  line   5.    Palpable  parasternally  but  retract  in  systole     6  to  20  choices:   nd th a.  2  Left  ICS     d.  4  ICS  PS   nd th b.  2  Right  ICS     e.  5  ICS  MCL   c.  Erb’s  point     D   6.  Pulmonic  stenosis   C   7.  VSD   D   8.  ASD   A   9.  PDA   10.     11.  Aortic  valve  regurgitation   E   12.  Mitral  valve  regurgitation   D   13.  Mitral  prolapse   B   14.  Aortic  stenosis   E   15.  Mitral  stenosis   E   16.  Loud  S1   A   17.  Opening  snap  

  A   18.  Loud  P2   D   19.  Hypertrophic  cardiomyopathy   E   20.  LV  hypertrophy     Multiple  choice  ito  na  super  haba,  eto  yung  summary   21.  Mitral  stenosis  –  diastolic  rumbling   22.  Aortic  stenosis  –  transmitted  to  the  carotid   23.  Aortic  regurgitation  –  Diastolic  rumbling  murmur   24.  VSD  –  pansystolic     25  to  30  choices:   a.  Caravallo’s  sign     d.  Gallavardin’s   b.  Kussmauls       e.  Austin-­‐Flint   c.  (+)  Hepatojugular     f.  Machinery-­‐like     D   25.  Aortic  stenosis   A   26.  Tricuspid  regurgitation   F   27.  PDA   E   28.  Chronic  aortic  regurgitation   C   29.  CHF   B   30.  Constrictive  pericarditis     31  to  35  choices:   a.  Loud  P2     d.  Opening  snap   b.  Loud  A2     e.  Ejection  Click   c.  Non-­‐ejection  sound     D   31.  Mitral  stenosis   C   32.  Mitral  valve  prolapse   33.  Constrictive  pericarditis   A   34.  Pulmonary  hypertension   B   35.  Chronic  uncontrolled  hypertension     36  to  40  choices:   a.  Paradoxical   b.  Wide   c.  Loud   d.  Negative  hepatojugular  reflex     36.  Mitral  stenosis   37.  Hypertensive  urgency   38.  Pulmonary  hypertension   A   39.  Severe  aortic  stenosis   40.  Pulmonary  hypertension                        

66  

  41  to  45  choices:   a.  Pulsus  parvus  et  tardus   b.  Corrigan  pulse   c.  Pulsus  bisferiens   d.  Hepatojugular   e.  Neck  vein  distention  at  60˚     D   41.  Normal  PE  finding   E   42.  Pulmonary  edema   A   43.  Aortic  stenosis   C   44.  Combined  aortic  regurgitation  and  aortic  stenosis   45.  Chronic  aortic  regurgitation     46  to  50  choices:   a.  Ejection  systolic  murmur  best  heard  at   b.  Persistent  splitting   c.  Diastolic  murmur   d.  Rumbling   e.  Blowing     46.     A   47.  Aortic  stenosis   D   48.  Mitral  stenosis   49.Aortic  regurgitation   50.  Pulmonic  stenosis       CVS     A.Concentric   B.  RVH   C.  Kussmaul  Sign   D.  +Hepatojugular  Reflex   E.  LV  Dilatation                                            

67  

 

68  

1.  52  year  old  female  known  to  be  diabetic  and  hypertensive  patient  admitted  because  of  severe  dyspnea  (Orthopnea),   cardiomegaly  and  bilateral  basal  crackles.  Neck  vein  is  visible  and  become  more  distended  upon  application  of  pressure  in   the  liver     2.   27   year   old   male   complained   of   fever,   dyspnea   and   chest   pain   aggravated   by   breathing.   Apical   beat   is   markedly   displaced  to  midaxillary  region  with  distant  heart  sound.  The  patient  is  diagnosed  with  viral  cardiomyopathy.     3.   42   year   old   male   who   was   not   aware   that   he   was   hypertensive,   when   seen   in   OPD   his   BP   was   180/110   on   sitting   position.  Apex  beat  at  5th  ICS  LAAL  with  apical  beat  to  be  sustained  and  foreceful.     4.   36   year   old   female   school   teacher   with   history   of   frequent   sore   throat   was   admitted   because   of   pedal   edema,   esay   fatigability  and  DOE.  +Parasternal  retraction  during  systole  and  an  apical  diastolic  rumbling  murmur     5.   65   years   old   male   known   CAD   patient   status   post   bypass   graft   surgery   10   years   ago   is   now   suffering   from   fatigue,   dyspnea,  orthopnea,  edema  and  pulmonary  edema.  Apical  beat  is  hardly  palpable  at  6th  ICS.  Axillary  region  faintly  audible   heart  sounds  and  adynamic  precordium   *****     A.  Loud  S1   B.  Soft  S1   C.  Loud  A2   D.  Loud  P2     1.  Chronic  regurgitation   2.  Hypertensive  Urgency  with  BP  220/120   3.  Rheumatic  Mitral  Stenosis   4.  Pulmonary  edema  in  a  patient  with  CHF   5.  Chronic  mitral  regurgitation   *****     A.  Physiologic  Splitting  S2   B.  Persistent  Splitting  S2   C.  Paradoxical  Spilitting  S2   D.  Wide  Splitting  S2     1.  ASD   2.  Pulmonic  Stenosis   3.  Aortic  Stenosis   4.  Complete  RBBB   5.  LBBB   ****                              

 

69  

A.  Right  mitral  stenosis   B.  Right  aortic  with  mitral  regurgitation   C.  Congenital  bicuspid  aortic  stenosis   D.  Congenital  pulmonic  stenosis   E.  Right  aortic  regurgitation       1.  36  year  old  male  admitted  because  or  bilateral  pedal  edema  and  orthopnea.  P.E  revealed  a  hyperdynamic  apical  pulse   seen  adn  palpable  at  6th  ICS  LMA  region  and  a  diastolic  blowing  murmur  at  the  ERBS  point  transmitted  to  the  apex     2.  A  42  year  old  GRO  was  admitted  because  of  easy  fatigability  pedal  edema  orthopnea  and  PND  which  started  around  6   months  PTC  becoming  progressive.  P.E  revealed  diastolic  thrill  and  murmur  with  Loud  S1  and  opening  snap     3.  42  year  old  male  admitted  because  of  chest  pain  and  episode  of  syncope.  P.E  revealed  +ejection  systolic  murmur  at  2nd   ICS  RSB  transmitted  to  the  neck     4.  30  year  old  female  presents  a  persistent  spilitting  of  S2  with  expiratory  splitting  of  S2  at  the  2nd  ICS  LCB  ejection  systolic   murmur  at  the  same  time     5.   42   year   old   male   known   RHD   patient   was   admitted   because   of   severe   dyspnea,   jaundice,   ascites,   hepatomegaly,   P.E   revealed   apical   beat   markedly   displaced   at   the   6th   ICS   left   midaxillary   region   with   apical   lift,   apical   diastolic   blowing   murmur  transmitted  from  the  axilla  and  posterior  scapula     Significant  Clinical  Findings   1. Important  physical  examination  finding  in  CHF  –  (+)  hepatojugular  reflux   2. Systolic  ejection  murmur  of  chronic  severe  aortic  regurgitation  is  accompanied  by  which  sound  –  Austin-­‐Flint   murmur   3. Mid-­‐diastolic  murmur  heard  over  the  pulmmonic  area  is  ASD  is  accompanied  by  which  sound  –  Persistent   splitting  of  S2   4. Increased  loudness  of  holosystolic  murmur  of  tricuspid  regurgitation  upon  inspiration  –  Caravallo’s  sign   5. Heard  in  aortic  stenosis  –  Paradoxical  splitting  of  S2     Cardiomyopathies   th 6. Apex  beat  is  displaced  at  the  6  ICS  LAAL,  (+)  para  sterna  heave,  (+)  lift  –  Eccentric   7. 40  year  old  female,  with  apical  beat  that  retracts  during  systole  –  Right  Ventricle  Hypertrophy   8. Hardly  palpable  –  Left  ventricle  dilatation   th 9. 40  year  old  female  with  chronic  hypertension  complained  of  chest  pain,  PE  revealed  apex  beat  at  the  5  ICS  LMCL   with  a  (+)  apical  lift  –  Concentric   10. A  52  year  old,  Male,  chronic  hypertensive  and  DM2  complained  of  chest  pain  and  SOB  (DOB?),  PE  revealed  (+)   th murmur  of  AR  and  MR,  apex  beat  is  displaced  at  5  ICS  left  anteroaxillary  line  with  (+)  lift.  –  Eccentric       Abnormal  Sounds   11. Pulmonary  Hypertension  –  loud  P2   12. Mitral  stenosis  –  Loud  S1   13. Atrial  Fibrillation  –  Soft  S1   14. BP  220/120  –  loud  S1   15. Pulmonic  stenosis  –Paradoxical  splitting  of  S2   16. Sinus  Tachycardia  –  loud  S1   17. Chronic  aortic  regurgitation  –  Loud  A2   18. LBBB  –  Paradoxical  splitting  of  S2   19. Severe  aortic  stenosis  –  Loud  A1   20. Normal  PE  findings  –  splitting  of  S2  upon  inspiration          

 

70  

Abnormal  Pulses   21. Chronic  aortic  regurgitation  and  aortic  Stenosis-­‐  Pulsus  Bisfierens   22. Aortic  regurgitation  –  Corrigan’s  pulse   23. Constructive  pericarditis  –  Pulsus  Paradoxus   24. Severe  aortic  Stenosis  –  Pulsus  parvus  et  tardus   25. CHF  –  pulsus  alterans     Cases   26. Diastolic  blowing  murmur  –  aortic  regurgitation   th 27. 4  ICS  parasternal  line  –  Tricuspid  regugitation   28. Ejection  systolic  murmur  transmitted  to  carotid  –  aortic  stenosis   29. Ejection  systolic  murmur  loud  on  expiration  –  pulmonic  stenosis   30. Holosystolic  murmur  displaced  to  left  axilla,  diminished  S1  –  Mitral  regurgitation         1. Important  physical  examination  finding  in  CHF   +  Hepatojugular  reflux   2. Mid  diastolic  murmur  heard  over  the  pulmonic  area  in  ASD  is  accompanied  by  what  sound?   Persistent  splitting  of  S2   3. Loudness  of  holosystolic  murmur  of  tricuspid  regurgitation  upon  inspiration   Caravallo’s  Sign   th 4. Apex  beat  is  displaced  at  the  6  ICS  LAAL,  faint  heart  sound,  (+)  parasternal  heave,  (+)  lift.   Eccentric  Hypertrophy   5. 40  year  old  female,  with  apical  beat  that  retracts  during  systole   RVH   th 6. SOB,  apex  at  6  ICS  left  midaxillary  line..faint  heart  sound,  alcoholic  and  elicit  drug  use   Eccentric   th 7. 40  year  old  with  chronic  hypertension  complained  of  chest  pain.  PE  revealed  apex  beat  @  5  ICS  LMCL  with  a  (+)   apical  lift.   Concentric   8. A  52  year  old  Male,  chronic  hypertension  &  DM2  complained  of  chest  pain  and  SOB.  PE  revealed  (+)  murmur  of   th AR  &  MR.  Apex  beat  is  displaced  @  5  ICS  left  anteroaxillary  line  with  (+)  lift.   Eccentric   9. Pulmonary  hypertension   Loud  P2   10. Atrial  fibrillation     Soft  S1   11. BP  220/120   Loud  S1   12. RBBB  persistent     13. Sinus  Tachycardia   Loud  S1   14. Chronic  Aortic  Regurgitation   Soft  S1   15. LBBB   Paradoxical  splitting  of  S2   16. Severe  Aortic  Stenosis   Paradoxical  splitting  of  S2  

 

71   17. Normal  PE  findings   Split  S2  on  inspiration   18. Chronic  aortic  regurgitation  and  aortic  stenosis   Pulsus  bisfiriens   19. Severe  aortic  stenosis     20. CHF   Pulsus  Alterans  

  21. A  27  year  old  female,  worried  about  ECG  finding  of  RBBB.  Auscultation  revealed  presence  of  ejection  systolic   murmur  which  became  louder  during  forceful  expiration.   Pulmonic  stenosis   22. 65  year  old  patient  with  DM,  HPN,  has  a  holosystolic  murmur  displaced  to  the  left  axilla,  has  a  diminished  S1   Mitral  regurgitation   23. Machinery-­‐like  murmur   PDA   24. Greater  BP  in  the  upper  extremities  compared  to  the  lower  extremities   Coarctation  of  Aorta   25. Characteristics  peripheral  pulse  in  CHF   Pulsus  Alterans   26. DM  hypertensive  64  year  old  experienced  pain  in  right  calf  muscle  on  walking  for  20  mins  and  relived  by  rest   Intermittent  Claudication   27. PDA   nd 2  Left  ICS   28. Aortic  Stenosis   nd 2  Right  ICS   29. S3  &  S4   th 5  ICS  LMCL   30. Mitral  Regurgitation     31. Fixed  splitting  of  S2   ASD   32. Holosystolic  murmur  at  the  left  parasternal  area  are  transmitted  to  the  right  sternal  border   VSD   33. Sail  Sound   Ebstein’s  anomaly   34. BP  elevated  in  the  upper  extremities  markedly  lower  in  the  lower  extremities   Coarctation  of  Aorta   35. Machinery-­‐like  murmur   PDA   36. Pulmonary  Stenosis   nd 2  Left  ICS              

  Clinical  Medicine  :  CARDIOVASCULAR  SYSTEM  EXAM   For  1  to  5,  choices  are:   a.  concentric       d.  RVH   b.  eccentric       e.  LVH   c.  LV  dilataion     1. Not  palpable  in  PE      C   2. Forceful  apical  beat  but  not  displaced.      A   3. Markedly  displaced  at  the  left  axillary  line,  hardly  palpable.    C   4. Displaced  to  axillary  line  5th  or  6th  ICS  anterior  axillary.      B   5. Palpable  parasternally  but  retract  in  systole.      D     For  6  to  20,  choices  are:   a.  2nd  LEFT  ICS     d.  4th  ICS  PS   b.  2nd  RIGHT  ICS   e.  5th  ICS  MCL   c.  Erb's  point     6. Pulmonic  Stenosis    A   7. VSD      C   8.     9.     10.     11. Aortic  valve  regurgitation      A  or  D   12. Mitral  valve  regurgitation      E   13. Mitral  Prolapse      E   14. Aortic  stenosis      B   15. Mitral  stenosis      E   16. Loud  S1      E   17. Opening  snap      A   18. Loud  P2      A   19. Hypertrophic  Cardiomyopathy      D   20. LV  Hypertrophy      E     multiple  choice  ito  na  super  haba,  eto  yung  summary:   21. Mitral  stenosis  -­‐  diastolic  rumbling   22. Aortic  stenosis  -­‐  transmitted  to  the  carotid   23. Aortic  Regurgitation  -­‐  diastolic  rumbling  murmur   24. VSD  -­‐  pansystolic     For  25  to  30,  choices  are:   a.  Caravallo's     d.  Gallavardin's   b.  Kussmaul's     e.  Austin-­‐Flint   c.  +  hepatojugular     f.  Machinery-­‐like     25. aortic  stenosis      D   26. tricuspid  regurgitation      A   27. PDA      F   28. chronic  aortic  regurgitation      E  

72  

  29. CHF      C   30. constrictive  pericarditis      B     For  31  to  35,  choices  are:   a.  loud  P2     d.  opening  snap   b.  loud  A2     e.  ejection  click   c.  non-­‐ejection  sound   31. Mitral  stenosis      D   32. Mitral  valve  prolapse      C   33. Constrictive  pericarditis      E   34. Pulmonary  Hypertension      A   35. Chronic  uncontrolled  hypertension      B     For  36  to  40,  choices  are:   a.  Paradoxical   b.  Wide   c.  Loud   d.  Negative  hepatojugular  reflex   36. Mitral  stenosis    C  (Loud  S2)   37. Hypertensive  urgency   38. Pulmonary  hypertension      C  (Loud  S2)   39. Severe  aortic  stenosis      A  (Paradoxical  splitting  of  S2)   40. Pulmonary  hypertension      C  (Loud  S2)     For  41  to  45,  choices  are  :   a.  Pulsus  parvus  et  tardus     d.  negative  hepatojugular   b.  Corrigan's  pulse       e.  neck  vein  distention  at  60  degrees   c.  pulsus  bisferiens   41. normal  PE  finding      D   42. Pulmonary  edema      E   43. Aortic  stenosis      A   44. Combined  aortic  regurgitation  and  aortic  stenosis      C   45. Chronic  aortic  regurgitation      B     For  46  to  50,  choices  are:   a.  ejection  systolic  murmur  best  heard  at   b.  persistent  splitting   c.  diastolic  murmur   d.  rumbling   e.  blowing   46. Mitral  regurgitation      D   47. Aortic  stenosis      A   48. Mitral  stenosis      C   49. Aortic  stenosis      E   50. Pulmonic  stenosis      B        

73  

 

74   1.

2.

3.

4. 5. 6. 7. 8. 9. 10.

11. 12. 13. 14. 15. 16. 17. 18. 19. 20.

The  anterior  portion  of  the  heart   a. LV     b. RV   c. Aorta   d. Pulmonary  Artery     The  diameter  of  the  apical  beat   a. 1.5cm   b. 2.5cm   c. 2.0cm   d. .5cm     Patient  with  COPD  has  palpable  pulse  at  the  epigastric  and  subxiphoid  region   a. Abdominal  aorta   b. RVH   c. LVD   d. Biventral  hypertrophy     Mitral  stenosis           a.  loud  s1   CAR             b.  soft  s1   Atrial  fibrillation           c.  loud  s2   MSD             d.  ejection                   a.  deep  inspiration                 b.  forceful  expiration   MVP             c.  leaning  forward   d  .LL  decubitous     Einsteins  Anomaly         a.  Caravalla’s   Chronic  Tricuspid  Regurgitation       b.  Galavardin’s   Aortic  Stenosis           c.  Snail  sound   Chronic  Severe  Aortic  Regurgitatin       d.  Paradox  pulse     nd Opening  snap  at  rheumatic  mitral  stenosis     a.  2  LICS   nd Pulmonic  regurgitation         b.  2  RICS   VSD             c.  Parasternal   MVP             d.  Erb’s   Congenital  Pulmonic  Stenosis       e.  Apex     Position  of  the  apical  beat  during  physical  exam   a. upright,  leaning  forward   b. supine   c. LLD            

 

75   21. Normal  location  of  the  apical  beat   a. Strong  and  forceful   th b. Always  at  5  ICP,  L  MCL   c. Gentle  tap   d. Strong  during  inspiration     22. Concentric  LV       23. Ischemic  Dilated  Cardiomyopathy   24. Chronic  Severe  Aortic  Stenosis   25. ASD         26. Normal    

       

       

a.  Persistent  S2   b.  Paradox  S2   c.  Physiologic  S2   d.    

    1. 2. 3. 4. 5. 6. 7. 8. 9. 10. 11. 12. 13. 14. 15. 16. 17. 18.        

Ventricular  Septal  Defect     >  HEARD  @  APEX   Very  loud  S1     >  BEST  HEARD  ON  APEX   Opening  Snap     >  BEST  HEARD  ON  PULMONIC  AREA   Apical  mid-­‐diastolic  rumbling  murmur     >  BEST  HEARD  ON  APEX   Chronic  Aortic  Regurgitation    >  ERBS  POINT   Acute  Severe  Mitral  Regurgitation  w/  s3  Gallop   RD  >  3  LICS   Right–Bundle  Branch  Block     >  PERSISTENT  /  WIDE  S2  SPLITTING   Left–Bundle  Branch  Block     >  PARADOXICAL  S2  SPLITTING   Non-­‐Systolic  Click     >  (MVP)  ERB’S  POINT     Less  Prolapse:  SQUAT  ;  More  Prolapse:  STAND   Ejection  Systolic  Murmur     (HEARD  LOUDEST  @  BASE)   Atrial  Septal  defect     ND >  2  LICS   Concentric  L-­‐ventricular  Hypertrophy   th >  5  ICS  LAAL   Restrictive  Hypertrophy  Cardiomyopathy     >  STANDING  –SQUATTING  POSISTION   Kussmaul  Sign     >  CAUSE  BY  INABILITY  OF  THE  (R)  HEART  TO  ACCOMODATE  INCREASE  VENOUSE  RETURN   Loud/accentuated  S1     >  HYPERTENSION   Physiologic  Splitting     >  2-­‐3  (L)  ICS   Paradoxical  Splitting  of  S2   >  (L)  BUNDLE  BRANCHING  BLOCK  

 

76  

  ABDOMEN     36-­‐40:  Draw  and  label  the  4  quadrants  of  the  abdomen  and  its  landmarks:   40-­‐50:  Draw  and  label  the  nine  regions  of  the  abdomen  and  its  landmarks:       Abdomen  –  Finals  Samplex   1. A  palpable  left  flank  mass  is  probably  the  left  kidney  if:   a. You’re  palpating  fingers  can  probe  deep  to  the  medial  and  lateral  borders  (splenomegaly  to)   b. Preservation  of  normal  tympany  in  LUQ   c. A  notch  is  palpated  on  medial  border  (splenomegaly  din)   d. Edge  of  mass  extend  beyond  the  midline   2. An  extremely  tight  sphincter  tone  on  rectal  examination  is  due  to:   a. Neuropathy  (seen  in  lax  sphincter  to)   c.  Cerebrovascular  disease   b. Spinal  cord  lesion       d.  Anxiety   3. A  35  y/o  male  complains  of  sudden  severe  epigastric  pain  radiating  to  the  back,  duration  noted  after  a  heavy   meal.  If  this  is  acute  abdominal  pain,  the  following  statement  is/are  correct   a. Always  mandate  surgical  intervention   c.  Can  be  manage  medially   b. Duration  of  pain  is  less  than  2  days     d.  Consider  if  the  pain  is  severe   4. A  21  y/o  male  with  acute  leukemia  have  a  palpable  splenic  notch.  The  traube’s  percussion  will  be:   a. Hyperresonant         c.  Resonant   b. Dull             d.  Tympanic   5. A  60  y/o  diabetic  had  stroke  1  month  ago  with  right  sided  hemiparesis  and  dysarthria..for  sensation  of  food  won’t   go  down  and  just  stay  on  the  mouth  and  repeatedly  attempts  to  swallow.  This  type  of  dysphagia  is:   a. Oropharyngeal         c.  Esophageal  dysphagia   b. Esophageal  web   6. In  patient  complaining  of  burning  sensation  that  begins  inferiorly  and  radiates  up  to  the  entire  retrosternal  area   to  the  neck  should  avoid  the  following  except:   a. NSAIDs       c.  Meperidine     e.  Citrus  food   b. Coffee       d.  Beta  blockers     ______________________________________________________________________________________     Abnormal  contour  –  xymphoid  to  symphisis  pubis   Portal  HPN:Except  -­‐  SMJ  nodule   Puddle  sign  –  flicking  sound  becomes  louder,  as  the  stet  goes  farther   Hypoactive  bowel  –  Ruber  test   Abdominal  paradox  –  chest  in,  abdomen  expand  upon  inspiration   Ovarian  cyst  and  ascites  –  hypokalemia,  furosemide,  abdominal  distention   Difference  of  voluntary  form  involuntary  rigidity  –  Inspire  with  wide  open  mouth   Carnette’s  sign  –  Abdominal  vs  Intramural  tendencies   Normal  liver  span  –  4-­‐8  cm  Midsternal,  6-­‐12  RMCL   Normal  size  of  aorta  –  3  cm   To  palpate  liver:except  –  Castell  technique  (splenomegaly)   th

Kidney  punch  –  12  rib,  costovertebral  angle   Methods  to  assess  Appx:  except  –  Boa’s  sign  

  Sim’s  position  –  Left  knee  flex,  side  of  the  table   Rectal  exam  in  women  –  mass,  tenderness,  nodules,  lateral  wall,  uterus   Acute  prostatitis  –  Boddy  and  tender   Grade  3  –  3-­‐4  cm  protrusion   Murphy’s  sign  –  Inspiratory  arrest   Using  Ulnar  surface  of  the  pt  hand  at  midline  –  fluid  wave   _____________________________________________________________________________________     Alcoholic  portal  hpn  massive  ascites  except:   a. shifting  dullness         c.  everted  umbilicus   b. fluid  wave         d.  Typhanitic     Nixon  –  lateral  decubitus     To  confirm  secondary  hpn   a. Epigastric       c.  R/L  iliac     e.  All   b. R/L  upper  q       d.  A  and  B     Obese  pt   a. light  palpate         c.  Reinforced  palpate   b. deep  palpate         d.  Ballotment           ABDOMEN   1. Abdominal  contour    →  Xyphoid  to  symphysis  pubis   2. Portal  Hypertension  →  Except:  SMJ  nodule   3. Puddle  Sign  →  Flicking  sound  becomes  louder,  as  the  stet  goes  farther   4. Hypoactive  bowel  →  Hypokalemia,  Furosemide,  Abdominal  distention   5. Ovarian  cyst  and  ascites  →  Ruler  Test   6. Differentiate  voluntary  to  involuntary  rigidity  →  Inspire  with  wide  open  mouth   7. Garnett’s  Sign  →  Abdominal  versus  intramural  tenderness   8. Normal  liver  span  →  4-­‐8  cm  Midsternal;  6-­‐12  RMCL   9. Normal  size  of  the  aorta  →  3cm   10. To  palpate  liver  →  Except:  Castell’s  technique  (for  splenomegaly)   th 11. Kidney  punch  →  12  rib,  costovertebral  angle   12. Method  to  assess  AP  (Appendicitis)  →  Except:  Boa’s  Sign   13. Sim’s  Position  →  Left  knee  flexed  side  of  the  table   14. Rectal  exam  in  women  →  Mass,  tenderness,  nodules,  lateral  wall,  uterus   15. Acute  prostatitis  →  Boggy  and  tender   16. Grade  3  →  3-­‐4cm  protrusion   17. Murphy’s  sign  →  Inspiratory  arrest   18. Using  ulnar  surface  of  patient  hand  in  the  midline  →  Fluid  wave      

77  

 

78  

  ABDOMEN   1. In  doing  fluid  wave,  the  ulnar  surface  of  the  patient’s  hand  is  pressed  into  the  midline  of  the  abdomen  to   a. Prevent  movement  of  intestines   b. Direct  the  movement  of  the  wave  to  the  opposite  side   c. Block  the  movement  of  the  mesenteric  fat   d. Clearly  visualize  the  fluid  wave     2. A  60  yo  CHF  for  which  he  was  maintained  on  Furosemide  developed  hypokalemia.  He  has  abdominal  distention   without  abdominal  pain.  The  auscultatory  findings  will  be   a. Succusion  splash   b. Hyperactive  bowel  sounds   c. Hypoactive  bowel  sounds   d. Normal  bowel  sounds     3. A  smoker  with  emphysema  was  noted  to  have  palpable  liver  3  finger  breaths  below  the  right  subcostal  margin.   The  expected  liver  span  is   a. 4-­‐8  cm  midsternal   b. 6-­‐12  MCL   c. 8-­‐14cm  anterior  axillary  line   d. All     4. All  but  one  are  expected  findings  in  a  46  yo  male  alcoholic  with  portal  hypertension     (All  are  expected  findings  in  a  46  yo  male  alcoholic  with  portal  hypertension  EXCEPT)   a. Globular  abdomen  with  everted  umbilicus   b. Sister  Mary  Joseph  nodules   c. Spider  angioma   d. Obliterated  Traube’s  space   e. (+)  Castells’s  sign  percussion     5. A  19  yo  male  have  severe  attacks  of  bronchial  asthma  was  noted  to  have  abdominal  paradox.  The  abdominal   respiratory  motion  is   a. Abdomen  contract  with  expiration   b. Rocking  motion  of  chest  and  abdomen   c. Abdomen  expands  while  chest  is  pulled  inward   d. All     6. To  differentiate  ascites  from  large  ovarian  cyst,  you  must  do   a. Ballotment  palpation   b. Deep  palpation   c. Shifting  dullness   d. Fluid  wave   e. Ruler  Test            

 

79   7.

8.

9.

10.

11.

12.

13.

Carnett’s  sign  will  differentiate  (please  double  check  na  lang  po  yung  correct  answer,  thanks!)   a. Intraabdominal  mass  from  intramural  mass   b. Rebound  tenderness  from  direct  tenderness   c. Intraabdominal  tenderness  from  abdominal  tenderness   d. Subcutaneous  crepitus  from  cutaneous  hyperthesia   A  45  yo  female  complaining  of  RUQ  pain  was  diagnosed  to  have  acute  cholecystitis.  A  positive  Murphy’s  sign  is   a. Tenderness  on  RUQ   b. Inspiratory  arrest   c. (+)  fist  tenderness  on  percussion  of  right  subcostal  margin   d. Exaggerated  pain  on  gentle  lifting  of  a  fold  of  skin  on  RUQ     In  a  22  yo  male  with  RLQ  pain  suspected  to  have  acute  appendicitis,  the  ff  are  positive  abdominal  signs,  EXCEPT   a. Markle’s   b. Blumberg’s   c. Aaron’s   d. Boa’s     e. Obturator     Positive  puddle’s  sign  is   a. Distinct  tap  on  your  palpating  hand   b. The  sound  becomes  louder  while  the  stethoscope  moves  away  from  the  flicking  spot   c. Dullness  shift  to  the  dependent  side  while  tympanitic  shift  to  the  top   d. A  visible  movement  as  a  tap  was  done  on  the  other  side  of  the  puddle     In  doing  the  kidney  punch,  a  direct  percussion  with  the  fist  should  be  applied  to   a. Subcostal  margin   b. Flank   th c. 12  rib  and  vertebral  angle   d. ASIS     A  66  yo  male  with  enlarged  prostate  Grade  III  on  rectal  examination,  have  this  amount  of  protrusion   a. 1-­‐2  cm   b. >2-­‐3  cm   c. >3-­‐4  cm   d. >4  cm       A  30  yo  male  complaining  of  dysuria  and  fever  with  pyuria  on  urinalysis.  The  expected  rectal  examination  findings   if  this  is  acute  bacterial  prostatitis  is/are   a. Sulcus  is  obliterated   b. >1  cm  protrusion   c. Nodular   d. Boggy  and  tender   e. All            

 

80   14. The  ff  statements  is/are  true  with  regards  to  significance  of  rectal  examination  in  female  EXCEPT   a. Cervix  may  be  palpable  through  anterior  wallt   b. Uterus  is  never  palpable  on  rectal  examination   c. Tenderness  of  peritoneal  inflammation  can  be  appreciated   d. Nodularity  of  peritoneal  metastasis  can  be  felt     15. Sequence  of  abdominal  examination   ANS.  Inspection,  Auscultation,  Palpation,  Percussion,  Special  Examination   16. Reference  used  in  determining  abdominal  contour  is  an  imaginary  line  drawn  from   a. Breast  to  umbilicus   b. Clavicle  to  the  symphysis  pubis   c. Rib  margin  to  umbilicus   d. Xiphoid  or  rib  margin  to  the  pubis     17. To  differentiate  between  abdominal  rigidity  amd  voluntary  muscle  guarding,  examiner  should  do  this  while  doing   palpation   a. Distract  the  patient  by  conversation   b. Ask  the  patient  to  breathe  with  mouth  wide  open  and  feel  relaxation  of  abdomen  during  expiration   c. Ask  patient  to  raise  his  head  from  supine  position   d. All   e. A  and  B  only     18. Technique  to  palpate  a  large  organ  is  freely  movable  as  mass  obscured  by  ascites   a. Capture  technique   b. Deep  palpation   c. Reinforced  palpation   d. Ballotment     19. Sim’s  position  of  patient  in  rectal  examination   a. Bent  over  the  examining  table   b. Lateral  position  lying  on  the  left  side  with  right  hip  and  knee  flexed  with  buttocks  close  to  edge  of  table     c. Knee-­‐chest  position   d. Lithotomy  position     20. An  enlarged  prostate  gland  on  rectal  examination  has  the  following  findings  EXCEPT   a. Firm,  rubbery  consistency   b. Obliteration  of  sulcus   c. >1cm  protrusion  into  the  rectum   d. Lateral  lobes  are  palpable  

                 

  Abdomen   A.MATCHING  TYPE     D1.Ovarian  cyst  vs  Ascites   A2.Obesity  vs.  Ascites   B3.Intramural  vs.  Intraabdominal  mass   C4.Peritonitis  vs.  Organomegaly   E5.-­‐-­‐-­‐-­‐-­‐-­‐-­‐-­‐-­‐-­‐-­‐-­‐-­‐-­‐-­‐-­‐   A.  Inverted  umbilicus   B.  Lift  head  while  in  supine  position   C.  Carnette’s  sign   D.  Ruler  test   E.  Traube’s  space   B.MULTIPLE  CHOICE   6.Visceral  pain  has  the  following  characteristics          A.  More  intense  and  more  localized          B.  Patient  move  about  in  an  effort  to  relieve  the  discomfort          C.  Aggrevated  by  moving  or  coughing          D.  Felt  in  areas  remote  to  diseased  organ   7.Acute  abdominal  pain,except          A.  Approximately  3cm     b.  direction  of  aortic  pulsation  is  directly  on  the  palpating  fingers     c.  lateral  pulsation  of  aorta  is  noted     d.  all     e.  a  &  b  only       10.  An  80y/o  male,  developed  anuria  &  complaining  of  hypogastric  pain.  Rectal  examination  revealed  grade  4  prostate   enlargement.  The  expected  abdominal  finding/s  is/are   a.  palpable  mass  on  hypogastric  area       d.  all   b.  dullness  on  percussion  of  hypogastric  area       e.  a  &  b   c.  symmetrically  globular  abdomen      

  11.  Normal  prostate  gland  on  rectal  examination  has  the  following  findings,  except     a.  firm  and  rubbery  in  consistency     c.  nontender  and  movable     b.  prominent  median  sulcus     d.  5cm  diameter  with  >1cm  protrusion     12.  On  rectal  examination  the  examining  finger  can  palpate  a  distance  of     a.  2-­‐4  cm     b.  4-­‐6  cm   c.  6-­‐10  cm   d.  8-­‐12cm     13.  On  PE  of  the  abdomen,  auscultation  is  done  before  doing  palpation  because:     a.  it  will  prevent  ticklishness  of  the  patient     b.  palpation  alters  frequency  of  bowel  sounds     c.  more  convenient  for  the  examiner     d.  it  is  more  preferred  by  the  patient     14.  Reference  used  in  determining  the  abdominal  contour  is  an  imaginary  line  drawn  from:     a.  rib  margin  to  the  umbilicus     c.  xiphoid  to  rib  margin  to  the  symphysis  pubis     b.  breast  to  the  umbilicus       d.  clavicle  to  the  symphysis  pubis     15.  All  but  one  are  use  to  overcome  ticklishness  of  the  patient  on  palpation  of  the  abdomen,  EXCEPT     a.  ask  patient  to  perform  self-­‐palpation     b.  talk  to  the  patient     c.  place  your  hands  over  patients  finger  and  after  sometime  drift  slowly  your  fingers  onto  the  abdomen     d.  use  diaphragm  of  stethoscope  as  palpating  instrument                                                                      

89  

 

90  

ABDOMEN     1.  PARIETAL  PAIN  -­‐  More  intense  and  more  precisely  located  -­‐  Aggravated  by  movement  or  coughing     2.  ACUTE  ABDOMINAL  PAIN  -­‐  1L)     8.  OCCULT  GI  BLEEDING  -­‐  Chronic  anemia  -­‐  No  change  in  color  -­‐  (+)  Guiac’s  test     9.  BLEEDING  OF  OBSCURE  ORIGIN  -­‐  All  of  the  above     10.  PEPTIC  ULCER  -­‐  Upper  GI  bleeding  or  massive  bleeding     11.  AMOEBIC  COLITIS  -­‐  Small  painful  stool  (tenesmus)     12.  ALLERGY  IN  CRABS  AND  OYSTERS  -­‐  Vibrio  species,  salmonella,  hepatitis  A     13.  All  are  part  of  ROME  2  CRITERIA  except:  -­‐  Lumpy  or  hard  stool  -­‐  Sensation  of  incomplete  evacuation  -­‐  bowel  action  per   week     14.  CONSTIPATION  IN  ELDERLY  -­‐  Decrease  food  intake  -­‐  Weak  abdominal  and  pelvic  muscles  -­‐  Slow  colonic  transit     15.  HESITANCY  -­‐  Delay  between  attempting  to  initiate  urination  and  actual  flow  of  urine     16.  CAUSE  OF  PYURIA  -­‐  All  of  the  above     17.  OLIGURIA  -­‐  
View more...

Comments

Copyright ©2017 KUPDF Inc.
SUPPORT KUPDF